Olds Maternal-Newborn Nursing Ch 34, 33, 26, 25, 23, 22, 21, 20

Pataasin ang iyong marka sa homework at exams ngayon gamit ang Quizwiz!

Which of the following is a common barbiturate used in labor? 1. Seconal 2. Valium 3. Phenergan 4. Vistaril

1. Seconal

A woman is admitted to the birth setting in early labor. She is 3 cm dilated, -2 station, with intact membranes and FHR of 150 beats/min. Her membranes rupture spontaneously, and the FHR drops to 90 beats/min with variable decelerations. What would the initial response from the nurse be? 1. Perform a vaginal exam. 2. Notify the physician. 3. Place the client in a left lateral position. 4. Administer oxygen at 2 L per nasal cannula.

Answer: 1 Explanation: 1. A drop in fetal heart rate accompanied by variable decelerations is consistent with a prolapsed cord. The nurse would assess for prolapsed cord via vaginal examination.

The nurse auscultates the FHR and determines a rate of 112 beats/min. Which action is appropriate? 1. Inform the maternal client that the rate is normal. 2. Reassess the FHR in 5 minutes because the rate is low. 3. Report the FHR to the doctor immediately. 4. Turn the maternal client on her side and administer oxygen.

Answer: 1 Explanation: 1. A fetal heart rate of 112 beats/min. falls within the normal range of 110-160 beats/min., so there is no need to inform the doctor.

A cesarean section is ordered for a pregnant client. Because the client is to receive general anesthesia, what is the primary danger with which the nurse is concerned? 1. Fetal depression 2. Vomiting 3. Maternal depression 4. Uterine relaxation

Answer: 1 Explanation: 1. A primary danger of general anesthesia is fetal depression. The depression in the fetus is directly proportional to the depth and duration of the anesthesia.

The nurse is making client assignments for the next shift. Which client is most likely to experience a complicated labor pattern? 1. 34-year-old woman at 39 weeks' gestation with a large-for-gestational-age (LGA) fetus 2. 22-year-old woman at 23 weeks' gestation with ruptured membranes 3. 30-year-old woman at 41 weeks' gestation and estimated fetal weight 7 pounds 8 ounces 4. 43-year-old woman at 37 weeks' gestation with hypertension

Answer: 1 Explanation: 1. A risk factor for hypotonic uterine contraction patterns includes a large-for-gestational-age (LGA) fetus.

How would the nurse best analyze the results from a client's sonogram that shows the fetal shoulder as the presenting part? 1. Breech, transverse 2. Breech, longitudinal 3. Breech, frank 4. Vertex, transverse

Answer: 1 Explanation: 1. A shoulder presentation is one type of breech presentation, and is also called a transverse lie.

The nurse is performing discharge teaching for a newly delivered first-time mother and her infant on the 2nd postpartum day. Which statement by the mother indicates that teaching has been successful? 1. "Taking baths will help my perineum feel less sore each day." 2. "If I develop heavy bleeding, I should take my temperature." 3. "My bowel movements should resume in a week." 4. "I will go back to the doctor in 4 days for my RhoGAM shot."

Answer: 1 Explanation: 1. A sitz bath or tub bath promotes healing and provides relief from perineal discomfort during the initial weeks following birth.

The nurse has just palpated contractions and compares the consistency to that of the forehead to estimate the firmness of the fundus. What would the intensity of these contractions be identified as? 1. Mild 2. Moderate 3. Strong 4. Weak

Answer: 3 Explanation: 3. The consistency of strong contractions is similar to that of the forehead.

The charge nurse has received the shift change report. Which client requires immediate intervention? 1. Woman at 6 cm undergoing induction of labor, strong contractions every 3 minutes 2. Woman at 4 cm whose fetus is in a longitudinal lie with a cephalic presentation 3. Woman at 10 cm and fetus at +2 station experiencing a strong expulsion urge 4. Woman at 3 cm screaming in fear because her mother died during childbirth

Answer: 4 Explanation: 4. This client is most likely fearful that she will die during labor because her mother died during childbirth. This client requires education and a great deal of support, and is therefore the top priority.

The multiparous client at term has arrived to the labor and delivery unit in active labor with intact membranes. Leopold maneuvers indicate the fetus is in a transverse lie with a shoulder presentation. Which physician order is most important? 1. Artificially rupture membranes. 2. Apply internal fetal scalp electrode. 3. Monitor maternal blood pressure every 15 minutes. 4. Alert surgical team of urgent cesarean.

Answer: 4 Explanation: 4. This is the highest priority because vaginal birth is impossible with a transverse lie. Labor should not be allowed to continue, and a cesarean birth is done quickly.

On assessment, a labor client is noted to have cardiovascular and respiratory collapse and is unresponsive. What should the nurse suspect? 1. An amniotic fluid embolus 2. Placental abruption 3. Placenta accreta 4. Retained placenta

answer 1 Explanation: 1. Cardiovascular and respiratory collapse are symptoms of an amniotic fluid embolus and cor pulmonale.

The charge nurse is looking at the charts of laboring clients. Which client is in greatest need of further intervention? 1. Woman at 7 cm, fetal heart tones auscultated every 90 minutes 2. Woman at 10 cm and pushing, external fetal monitor applied 3. Woman with meconium-stained fluid, internal fetal scalp electrode in use 4. Woman in preterm labor, external monitor in place

: 1 Explanation: 1. During active labor, the fetal heart tones should be auscultated every 30 minutes; every 90 minutes is not frequent enough.

Which client requires immediate intervention by the labor and delivery nurse? 1. Client at 8 cm, systolic blood pressure has increased 35 mm Hg 2. Client who delivered 1 hour ago with WBC of 50,000 3. Client at 5 cm with a respiratory rate of 22 between contractions 4. Client in active labor with polyuria

: 2 Explanation: 2. The white blood cell (WBC) count increases to 25,000/mm3 to 30,000/mm3 during labor and early postpartum. This count is abnormally high, and requires further assessment and provider notification.

A variety of drugs are used either alone or in combination to provide relief of postpartum pain. Which of the following would be an option for pain relief? 1. Nonsteroidal anti-inflammatory agents 2. Proquad 3. Methergine 4. Intravenous oxytocin

Answer: 1 Explanation: 1. A variety of drugs are used alone or in combination to provide relief of postpartum pain. An option would include nonsteroidal anti-inflammatory agents such as ibuprofen and ketorolac.

To reduce possible side effects from a cesarean section under general anesthesia, clients are routinely given which type of medication? 1. Antacids 2. Tranquilizers 3. Antihypertensives 4. Anticonvulsants

Answer: 1 Explanation: 1. Antacids are routinely administered before surgery for a cesarean section.

The nurse is scheduling a client for an external cephalic version (ECV). Which finding in the client's chart requires immediate intervention? 1. Previous birth by cesarean 2. Frank breech ballotable 3. 37 weeks, complete breech 4. Failed ECV last week

Answer: 1 Explanation: 1. Any previous uterine scar is a contraindication to ECV. Prior scarring of the uterus may increase the risk of uterine tearing or uterine rupture.

A nurse needs to evaluate the progress of a woman's labor. The nurse obtains the following data: cervical dilatation 6 cm; contractions mild in intensity, occurring every 5 minutes, with a duration of 30-40 seconds. Which clue in this data does not fit the pattern suggested by the rest of the clues? 1. Cervical dilatation 6 cm 2. Mild contraction intensity 3. Contraction frequency every 5 minutes 4. Contraction duration 30-40 seconds

Answer: 1 Explanation: 1. Cervical dilatation of 6 cm indicates the active phase of labor. During this phase the cervix dilates from about 4 to 7 cm and contractions and pain intensify.

A laboring client's obstetrician has suggested amniotomy as a method for creating stronger contractions and facilitating birth. The client asks, "What are the advantages of doing this?" What should the nurse cite in response? 1. Contractions elicited are similar to those of spontaneous labor. 2. Amniotomy decreases the chances of a prolapsed cord. 3. Amniotomy reduces the pain of labor and makes it easier to manage. 4. The client will not need an episiotomy.

Answer: 1 Explanation: 1. Contractions after amniotomy are similar to those of spontaneous labor.

What would be a normal cervical dilatation rate in a first-time mother ("primip")? 1. 1.5 cm per hour 2. Less than 1 cm cervical dilatation per hour 3. 1 cm per hour 4. Less than 0.5 cm per hour

Answer: 1 Explanation: 1. Dilatation in a "multip" is about 1.5 cm per hour.

The nurse is analyzing several fetal heart rate patterns. The pattern that would be of most concern to the nurse would be which of the following? 1. Moderate variability 2. Early decelerations 3. Late decelerations 4. Accelerations

Answer: 3 Explanation: 3. Late decelerations are caused by uteroplacental insufficiency. The late deceleration pattern is considered a nonreassuring sign.

After delivery, it is determined that there is a placenta accreta. Which intervention should the nurse anticipate? 1. 2 L oxygen by mask 2. Intravenous antibiotics 3. Intravenous oxytocin 4. Hysterectomy

Answer: 4 Explanation: 4. The primary complication of placenta accreta is maternal hemorrhage and failure of the placenta to separate following birth of the infant. An abdominal hysterectomy may be the necessary treatment, depending on the amount and depth of involvement.

Which of the following is a major side effect of butorphanol tartrate (Stadol)? 1. Blurred vision 2. Agitation 3. Feelings of dysphoria 4. Drowsiness

Answer: 3 Explanation: 3. Feelings of dysphoria are a major side effect of Stardol.

The nurse knows that a contraindication to the induction of labor is which of the following? 1. Placenta previa 2. Isoimmunization 3. Diabetes mellitus 4. Premature rupture of membranes

Answer: 1 Explanation: 1. Placenta previa is a contraindication to the induction of labor.

The client presents to the labor and delivery unit stating that her water broke 2 hours ago. Barring any abnormalities, how often would the nurse expect to take the client's temperature? 1. Every hour 2. Every 2 hours 3. Every 4 hours 4. Every shift

Answer: 3 Explanation: 3. Maternal temperature is taken every 4 hours unless it is above 37.5°C. If elevated, it is taken every hour.

The client vaginally delivers an infant that weighs 4750 g. Moderate shoulder dystocia occurred during the birth. During the initial assessment of this infant, what should the nurse look for? 1. Bell's palsy 2. Bradycardia 3. Erb palsy 4. Petechiae

Answer: 3 Explanation: 3. Macrosomic newborns should be inspected for cephalhematoma, Erb palsy, and fractured clavicles.

The nurse is assessing clients after delivery. For which client is early discharge at 24 hours after delivery appropriate? 1. Woman and baby who have had two successful breastfeedings 2. Woman who is bottle-feeding her infant and has not voided since delivery 3. Twins delivered at 35 weeks, bottle-feeding 4. Cesarean birth performed for fetal distress

Answer: 1 Explanation: 1. Early discharge may be advantageous if mother and baby are doing well, help is available for the mother at home, and the family and physician/CNM agree that both clients are healthy and ready for discharge. Feeding successfully is one of the physiologic needs of the infant and both mother and infant appear to be doing well.

A postpartum client has inflamed hemorrhoids. Which nursing intervention would be appropriate? 1. Encourage sitz baths. 2. Position the client in the supine position. 3. Avoid stool softeners. 4. Decrease fluid intake.

Answer: 1 Explanation: 1. Encouraging sitz baths is the correct approach because moist heat decreases inflammation and provides for comfort.

The client has been pushing for 3 hours, and the fetus is making a slow descent. The partner asks the nurse whether pushing for this long is normal. How should the nurse respond? 1. "Your baby is taking a little longer than average, but is making progress." 2. "First babies take a long time to be born. The next baby will be easier." 3. "The birth would go faster if you had taken prenatal classes and practiced." 4. "Every baby is different; there really are no norms for labor and birth."

Answer: 1 Explanation: 1. Establishing rapport and a trusting relationship and providing information that is true is best response.

A client who delivered 2 hours ago tells the nurse that she is exhausted and feels guilty because her friends told her how euphoric they felt after giving birth. How should the nurse respond? 1. "Everyone is different, and both responses are normal." 2. "Most mothers do feel euphoria; I don't know why you don't." 3. "It's good for me to know that because it might indicate a problem." 4. "Let me bring your baby to the nursery so that you can rest."

Answer: 1 Explanation: 1. Following birth, some women feel exhausted and in need of rest. Other women are euphoric and full of psychic energy, ready to retell their experience of birth repeatedly.

A nurse is checking the postpartum orders. The doctor has prescribed bed rest for 6-12 hours. The nurse knows this is an appropriate order if the client had which type of anesthesia? 1. Spinal 2. Pudendal 3. General 4. Epidural

Answer: 1 Explanation: 1. Following the birth, the woman may be kept flat. Although the effectiveness of the supine position to avoid headache following a spinal is controversial, the physician's orders may include lying flat for 6 to 12 hours.

The nurse is caring for a laboring client with thrombocytopenia. During labor, it is determined that the client requires a cesarean delivery. The nurse is preparing the client for surgery, and should instruct the client that the recommended method of anesthesia is which of the following? 1. General anesthesia 2. Epidural anesthesia 3. Spinal anesthesia 4. Regional anesthesia

Answer: 1 Explanation: 1. General anesthesia will be recommended. Women with thrombocytopenia should avoid regional blocks.

An analgesic medication has been administered intramuscularly to a client in labor. How would the nurse evaluate if the medication was effective? 1. The client dozes between contractions. 2. The client is moaning during contractions. 3. The contractions decrease in intensity. 4. The contractions decrease in frequency.

Answer: 1 Explanation: 1. If the client dozes between contractions, the analgesic is effective. Analgesics decrease discomfort and increase relaxation.

A client delivered 30 minutes ago. Which postpartal assessment finding would require close nursing attention? 1. A soaked perineal pad since the last 15-minute check 2. An edematous perineum 3. The client experiencing tremors 4. A fundus located at the umbilicus

Answer: 1 Explanation: 1. If the perineal pad becomes soaked in a 15-minute period or if blood pools under the buttocks, continuous observation is necessary. As long as the woman remains in bed during the first hour, bleeding should not exceed saturation of one pad.

The client gave birth to a 7 pound, 14 ounce female 30 minutes ago. The placenta has not yet delivered. Manual removal of the placenta is planned. What should the nurse prepare to do? 1. Start an IV of lactated Ringer's. 2. Apply anti-embolism stockings. 3. Bottle-feed the infant. 4. Send the placenta to pathology.

Answer: 1 Explanation: 1. In women who do not have an epidural in place, intravenous sedation may be required because of the discomfort caused by the procedure. An IV is necessary.

The nurse would expect a physician to prescribe which medication to a postpartum client with heavy bleeding and a boggy uterus? 1. Methylergonovine maleate (Methergine) 2. Rh immune globulin (RhoGAM) 3. Terbutaline (Brethine) 4. Docusate (Colace)

Answer: 1 Explanation: 1. Methylergonovine maleate is the drug used for the prevention and control of postpartum hemorrhage.

A woman in active labor is given nalbuphine hydrochloride (Nubain) 14 mg IV for pain relief. Half an hour later, her respirations are at 8 per minute. The physician would likely order which medication for this client? 1. Narcan 2. Reglan 3. Benadryl 4. Vistaril

Answer: 1 Explanation: 1. Narcan is useful for respiratory depression caused by nalbuphine (Nubain).

The nurse administered oxytocin 20 units at the time of placental delivery. Why was this primarily done? 1. To contract the uterus and minimize bleeding 2. To decrease breast milk production 3. To decrease maternal blood pressure 4. To increase maternal blood pressure

Answer: 1 Explanation: 1. Oxytocin is given to contract the uterus and minimize bleeding.

A woman has been having contractions since 4 a.m. At 8 a.m., her cervix is dilated to 5 cm. Contractions are frequent, and mild to moderate in intensity. Cephalopelvic disproportion (CPD) has been ruled out. After giving the mother some sedation so she can rest, what would the nurse anticipate preparing for? 1. Oxytocin induction of labor 2. Amnioinfusion 3. Increased intravenous infusion 4. Cesarean section

Answer: 1 Explanation: 1. Oxytocin is the drug of choice for labor augmentation or labor induction and may be administered as needed for hypotonic labor patterns.

The nurse is preparing to assess the fetus of a laboring client. Which assessment should the nurse perform first? 1. Perform Leopold maneuvers to determine fetal position. 2. Count the fetal heart rate between, during, and for 30 seconds following a uterine contraction (UC). 3. Dry the maternal abdomen before using the Doppler. 4. The diaphragm should be cooled before using the Doppler.

Answer: 1 Explanation: 1. Performing Leopold maneuvers is the first step.

The nurse is inducing the labor of a client with severe preeclampsia. As labor progresses, fetal intolerance of labor develops. The induction medication is turned off, and the client is prepared for cesarean birth. Which statement should the nurse include in her preoperative teaching? 1. "Because of your preeclampsia you are at higher risk for hypotension after an epidural anesthesia." 2. "Because of your preeclampsia you might develop hypertension after a spinal anesthesia." 3. "Because of your preeclampsia your baby might have decreased blood pressure after birth." 4. "Because of your preeclampsia

Answer: 1 Explanation: 1. Pregnancies complicated by preeclampsia are high-risk situations. The woman with mild preeclampsia usually may have the analgesia or anesthesia of choice, although the incidence of hypotension with epidural anesthesia is increased. If hypotension occurs with the epidural block, it provides further stress on an already compromised cardiovascular system.

The nurse is reviewing charts of clients who underwent cesarean births by request in the last two years. The hospital is attempting to decrease costs of maternity care. What findings contribute to increased health care costs in clients undergoing cesarean birth by request? 1. Increased abnormal placenta implantation in subsequent pregnancies 2. Decreased use of general anesthesia with greater use of epidural anesthesia 3. Prolonged anemia, requiring blood transfusions every few months 4. Coordination of career projects of both partners leading to increased income

Answer: 1 Explanation: 1. Repeat cesarean births are associated with greater risks including increased incidence of abnormal placentation in subsequent pregnancies and the increased risk of mortality secondary to surgery, which would contribute to increased health care costs.

The community nurse is working with a client from Southeast Asia who has delivered her first child. Her mother has come to live with the family for several months. The nurse understands that the main role of the grandmother while visiting is to do which of the following? 1. Help the new mother by allowing her to focus on resting and caring for the baby. 2. Teach her son-in-law the right way to be a father because this is his first child. 3. Make sure that her daughter does not become abusive towards the infant. 4. Pass on the cultural values and beliefs to the newborn grandchild.

Answer: 1 Explanation: 1. Rest, seclusion, and dietary restraint practices in many traditional non-Western cultures (South Asian groups) are designed to assist the woman and her baby during postpartum vulnerable periods.

Which of the following conditions would predispose a client for thrombophlebitis? 1. Severe anemia 2. Cesarean delivery 3. Anorexia 4. Hypocoagulability

Answer: 1 Explanation: 1. Severe anemia would predispose a client for thrombophlebitis.

The nurse is teaching a class on vaginal birth after cesarean (VBAC). Which statement by a participant indicates that additional information is needed? 1. "Because the scar on my belly goes down from my navel, I am not a candidate for a VBAC." 2. "My first baby was in a breech position, so for this pregnancy, I can try a VBAC if the baby is head-down." 3. "Because my hospital is so small and in a rural area, they won't let me attempt a VBAC." 4. "The rate of complications from VBAC is lower than the rate of complications from a cesarean."

Answer: 1 Explanation: 1. Skin incision is not indicative of uterine incision. Only the uterine incision is a factor in deciding whether VBAC is advisable. The classic vertical incision was commonly done in the past and is associated with increased risk of uterine rupture in subsequent pregnancies and labor.

The nurse assesses the postpartum client to have moderate lochia rubra with clots. Which nursing intervention would be appropriate? 1. Assess fundus and bladder status. 2. Catheterize the client. 3. Administer Methergine IM per order. 4. Contact the physician immediately.

Answer: 1 Explanation: 1. The amount, consistency, color, and odor of the lochia are monitored on an ongoing basis. Increased bleeding is most often related to uterine atony and responds to fundal massage, expression of any clots, and emptying the bladder.

The client tells the nurse that she has come to the hospital so that her baby's position can be changed. The nurse would begin to organize the supplies needed to perform which procedure? 1. A version 2. An amniotomy 3. Leopold maneuvers 4. A ballottement

Answer: 1 Explanation: 1. Version, or turning the fetus, is a procedure used to change the fetal presentation by abdominal or intrauterine manipulation.

Two hours ago, a client at 39 weeks' gestation was 3 cm dilated, 40% effaced, and +1 station. Frequency of contractions was every 5 minutes with duration 40 seconds and intensity 50 mmHg. The current assessment is 4 cm dilated, 40% effaced, and +1 station. Frequency of contractions is now every 3 minutes with 40-50 seconds' duration and intensity of 40 mmHg. What would the priority intervention be? 1. Begin oxytocin after assessing for CPD. 2. Give terbutaline to stop the preterm labor. 3. Start oxygen at 8 L/min. 4. Have the anesthesiologist give the client an epidural.

Answer: 1 Explanation: 1. The client is having hypertonic contractions. Cephalopelvic disproportion (CPD) must be excluded. If CPD exists, oxytocin (Pitocin) augmentation should not be used. Oxytocin is the drug of choice for labor augmentation or labor induction.

The laboring client's fetal heart rate baseline is 120 beats per minute. Accelerations are present to 135 beats/min. During contractions, the fetal heart rate gradually slows to 110, and is at 120 by the end of the contraction. What nursing action is best? 1. Document the fetal heart rate. 2. Apply oxygen via mask at 10 liters. 3. Prepare for imminent delivery. 4. Assist the client into Fowler's position.

Answer: 1 Explanation: 1. The described fetal heart rate has a normal baseline; the presence of accelerations indicates adequate fetal oxygenation, and early decelerations are normal. No intervention is necessary.

To identify the duration of a contraction, the nurse would do which of the following? 1. Start timing from the beginning of one contraction to the completion of the same contraction. 2. Time between the beginning of one contraction and the beginning of the next contraction. 3. Palpate for the strength of the contraction at its peak. 4. Time from the beginning of the contraction to the peak of the same contraction.

Answer: 1 Explanation: 1. The duration of each contraction is measured from the beginning of the contraction to the completion of the contraction.

The nurse is planning care for three newly delivered adolescents and their babies. What should the nurse keep in mind when planning their care? 1. The baby's father should be encouraged to participate when the nurse is providing instruction. 2. A class for all the adolescents would decrease teaching effectiveness. 3. The schools that the adolescents attend will provide teaching on bathing. 4. Adolescents understand the danger signals in newborns.

Answer: 1 Explanation: 1. The father, if he is involved, should be included as much as possible. If classes are offered in the hospital during the postpartum stay, the adolescent mother and father should be strongly encouraged to attend and participate.

The fetal heart rate baseline is 140 beats/min. When contractions begin, the fetal heart rate drops suddenly to 120, and rapidly returns to 140 before the end of the contraction. Which nursing intervention is best? 1. Assist the client to change position. 2. Apply oxygen to the client at 2 liters per nasal cannula. 3. Notify the operating room of the need for a cesarean birth. 4. Determine the color of the leaking amniotic fluid.

Answer: 1 Explanation: 1. The fetus is exhibiting variable decelerations, which are caused by cord compression. Sometimes late or variable decelerations are due to the supine position of the laboring woman. In this case, the decrease in uterine blood flow to the fetus may be alleviated by raising the woman's upper trunk or turning her to the side to displace pressure of the gravid uterus on the inferior vena cava.

What is the major adverse side effect of epidural anesthesia? 1. Maternal hypotension 2. Decrease in variability of the FHR 3. Vertigo 4. Decreased or absent respiratory movements

Answer: 1 Explanation: 1. The major adverse effect of epidural anesthesia is maternal hypotension caused by a spinal blockade, which lowers peripheral resistance, decreases venous return to the heart, and subsequently lessens cardiac output and lowers blood pressure.

After administration of an epidural anesthetic to a client in active labor, it is most important to assess the mother immediately for which of the following? 1. Hypotension 2. Headache 3. Urinary retention 4. Bradycardia

Answer: 1 Explanation: 1. The most common complication of an epidural is maternal hypotension.

What is one of the most common initial signs of nonreassuring fetal status? 1. Meconium-stained amniotic fluid 2. Cyanosis 3. Dehydration 4. Arrest of descent

Answer: 1 Explanation: 1. The most common initial signs of nonreassuring fetal status are meconium-stained amniotic fluid and changes in the fetal heart rate (FHR).

Which of the following behaviors noted in the postpartum client would require the nurse to assess further? 1. Responds hesitantly to infant cries. 2. Expresses satisfaction about the sex of the baby. 3. Friends and family visit the client and give advice. 4. Talks to and cuddles with the infant frequently.

Answer: 1 Explanation: 1. The mother tends to respond verbally to any sounds emitted by the newborn, such as cries, coughs, sneezes, and grunts. Responding hesitantly to infant cries might need further assessment to determine what the mother is feeling.

After being in labor for several hours with no progress, a client is diagnosed with CPD (cephalopelvic disproportion), and must have a cesarean section. The client is worried that she will not be able to have any future children vaginally. After sharing this information with her care provider, the nurse would anticipate that the client would receive what type of incision? 1. Transverse 2. Infraumbilical midline 3. Classic 4. Vertical

Answer: 1 Explanation: 1. The transverse incision is made across the lowest and narrowest part of the abdomen and is the most common lower uterine segment incision.

The laboring client is complaining of tingling and numbness in her fingers and toes, dizziness, and spots before her eyes. The nurse recognizes that these are clinical manifestations of which of the following? 1. Hyperventilation 2. Seizure auras 3. Imminent birth 4. Anxiety

Answer: 1 Explanation: 1. These symptoms all are consistent with hyperventilation.

By inquiring about the expectations and plans that a laboring woman and her partner have for the labor and birth, the nurse is primarily doing which of the following? 1. Recognizing the client as an active participant in her own care. 2. Attempting to correct any misinformation the client might have received. 3. Acting as an advocate for the client. 4. Establishing rapport with the client.

Answer: 1 Explanation: 1. Understanding the couple's expectations and plans helps the nurse provide optimal nursing care and facilitate the best possible birth experience.

The laboring client presses the call light and reports that her water has just broken. What would the nurse's first action be? 1. Check fetal heart tones. 2. Encourage the mother to go for a walk. 3. Change bed linens. 4. Call the physician.

Answer: 1 Explanation: 1. When the membranes rupture, the nurse notes the color and odor of the amniotic fluid and the time of rupture and immediately auscultates the FHR.

Two hours after delivery, a client's fundus is boggy and has risen to above the umbilicus. What is the first action the nurse would take? 1. Massage the fundus until firm 2. Express retained clots 3. Increase the intravenous solution 4. Call the physician

Answer: 1 Explanation: 1. When the uterus becomes boggy, pooling of blood occurs within it, resulting in the formation of clots. Anything left in the uterus prevents it from contracting effectively. Thus if it becomes boggy or appears to rise in the abdomen, the fundus should be massaged until firm.

The client has asked the nurse why her cervix has only changed from 1 to 2 cm in 3 hours of contractions occurring every 5 minutes. What is the nurse's best response to the client? 1. "Your cervix has also effaced, or thinned out, and that change in the cervix is also labor progress." 2. "When your perineal body thins out, your cervix will begin to dilate much faster than it is now." 3. "What did you expect? You've only had contractions for a few hours. Labor takes time." 4. "The hormones that cause labor to begin are just getting to be at levels that will change your cervix."

Answer: 1 Explanation: 1. With each contraction, the muscles of the upper uterine segment shorten and exert a longitudinal traction on the cervix, causing effacement. Effacement is the taking up (or drawing up) of the internal os and the cervical canal into the uterine side walls.

A 26-year-old client is having her initial prenatal appointment. The client reports to the nurse that she suffered a pelvic fracture in a car accident 3 years ago. The client asks whether her pelvic fracture might affect her ability to have a vaginal delivery. What response by the nurse is best? 1. "It depends on how your pelvis healed." 2. "You will need to have a cesarean birth." 3. "Please talk to your doctor about that." 4. "You will be able to delivery vaginally."

Answer: 1 Explanation: 1. Women with a history of pelvic fractures may also be at risk for cephalopelvic disproportion (CPD).

A client attending a prenatal class asks why episiotomies are performed. The nurse explains that risk factors that predispose women to episiotomies include which of the following? Note: Credit will be given only if all correct choices and no incorrect choices are selected. Select all that apply. 1. Large or macrosomic fetus 2. Use of forceps 3. Shoulder dystocia 4. Maternal health 5. Shorter second stage

Answer: 1, 2, 3 Explanation: 1. A large fetus places a woman at risk for an episiotomy to prevent lacerations. 2. Use of forceps or vacuum extractor is a risk factor that predisposes women to episiotomies. 3. Shoulder dystocia is a risk factor that predisposes women to episiotomies.

A cesarean section is ordered for the laboring client with whom the nurse has worked all shift. The client will receive general anesthesia. The nurse knows that potential complications of general anesthesia include which of the following? Note: Credit will be given only if all correct choices and no incorrect choices are selected. Select all that apply. 1. Fetal depression that is directly proportional to the depth and duration of the anesthesia 2. Poor fetal metabolism of anesthesia, which inhibits use with preterm infants 3. Uterine relaxation 4. Increased gastric motility 5. Itching of the face and neck

Answer: 1, 2, 3 Explanation: 1. A primary danger of general anesthesia is fetal depression, because the medication reaches the fetus in about 2 minutes. The depression is directly proportional to the depth and duration of anesthesia. 2. The poor fetal metabolism of general anesthetic agents is similar to that of analgesic agents administered during labor. General anesthesia is not advocated when the fetus is considered to be at high risk, particularly in preterm birth. 3. Most general anesthetic agents cause some degree of uterine relaxation.

The labor nurse would not encourage a mother to bear down until the cervix is completely dilated, to prevent which of the following? Note: Credit will be given only for all correct choices and no incorrect choices. Select all that apply. 1. Maternal exhaustion 2. Cervical edema 3. Tearing and bruising of the cervix 4. Enhanced perineal thinning 5. Having to perform an episiotomy

Answer: 1, 2, 3 Explanation: 1. If the cervix is not completely dilated, maternal exhaustion can occur. 2. If the cervix is not completely dilated, cervical edema can occur. 3. If the cervix is not completely dilated, tearing and bruising of the cervix can occur.

The nurse is caring for a client who could be at risk for uterine rupture. The nurse is monitoring the fetus closely for which of the following? Note: Credit will be given only if all correct choices and no incorrect choices are selected. Select all that apply. 1. Late decelerations 2. Bradycardia 3. Loss of ability to determine fetal station 4. Tachycardia 5. Early decelerations

Answer: 1, 2, 3 Explanation: 1. Late decelerations could be seen with uterine rupture. 2. Bradycardia is seen if there is uterine rupture. 3. The uterus is not holding the fetus in place anymore if the uterus ruptures.

The nurse caring for a client in labor anticipates fetal macrosomia and shoulder dystocia. Appropriate management of shoulder dystocia is essential in order to prevent which fetal complications? Note: Credit will be given only if all correct choices and no incorrect choices are selected. Select all that apply. 1. Brachial plexus injury 2. Fractured clavicle 3. Asphyxia 4. Neurological damage 5. Puerperal infection

Answer: 1, 2, 3, 4 Explanation: 1. Brachial plexus injury occurs due to improper or excessive traction applied to the fetal head. 2. Complications in macrosomia include fractured clavicles. 3. Complications in macrosomia include asphyxia of the fetus. 4. Neurological damage is a complication of macrosomia.

For what common side effects of epidural anesthesia should the nurse watch? Note: Credit will be given only if all correct choices and no incorrect choices are selected. Select all that apply. 1. Elevated maternal temperature 2. Urinary retention 3. Nausea 4. Long-term back pain 5. Local itching

Answer: 1, 2, 3, 5 Explanation: 1. Elevated maternal temperature is a potential side effect of epidural anesthesia. 2. Urinary retention is a potential side effect of epidural anesthesia. 3. Nausea is a potential side effect of epidural anesthesia. 5. Pruritus may occur at any time during the epidural infusion. It usually appears first on the face, neck, or torso and is generally the result of the agent used in the epidural infusion. Benadryl, an antihistamine, can be administered to manage pruritus.

The postpartum nurse provides anticipatory guidance for the new mother as well as teaching on self-care and infant care before discharge. Which topics should be included? Note: Credit will be given only if all correct choices and no incorrect choices are selected. Select all that apply. 1. Role changes brought on by the addition to the family unit 2. The realities of having a new baby, and how it affects previous lifestyle 3. Potential complications such as infant colic and postpartum issues 4. Sexuality and contraception 5. Toilet-training and preschool options

Answer: 1, 2, 3, 4 Explanation: 1. It is helpful for the nurse to advise parents that they may experience feelings of uncertainty as they grow into the parental role and alter their family processes to accommodate the new family member. 2. Guidance is essential in assisting the family to cope with role changes and the realities of a new baby. 3. Guidance is essential in assisting the family to cope with potential complications such as infant colic and postpartum health issues. 4. It is important for the nurse to present information about changes that may affect sexual activity and to discuss the importance of contraception during the early postpartum period.

The nurse is caring for a client who is having fetal tachycardia. The nurse knows that possible causes include which of the following? Note: Credit will be given only if all correct choices and no incorrect choices are selected. Select all that apply. 1. Maternal dehydration 2. Maternal hyperthyroidism 3. Fetal hypoxia 4. Prematurity 5. Anesthesia or regional analgesia

Answer: 1, 2, 3, 4 Explanation: 1. Maternal dehydration can cause fetal tachycardia. 2. Maternal hyperthyroidism can cause fetal tachycardia. 3. Fetal tachycardia can indicate fetal hypoxia. 4. Prematurity can cause fetal tachycardia.

The nurse is teaching a new mother about ways to manage fatigue after she returns home. Which instructions should the nurse include? Note: Credit will be given only if all correct choices and no incorrect choices are selected. Select all that apply. 1. Take frequent rest periods. 2. Nap when the newborn is sleeping. 3. Avoid overdoing housework and unnecessary chores. Do not clean when infant is sleeping. 4. Avoid having others come to the house to do housework and interfere with rest. 5. Utilize friends and family to provide help and support, such as cooking a meal.

Answer: 1, 2, 3, 5 Explanation: 1. Adequate rest is essential to a smooth postpartum transition. The nurse can encourage rest by organizing activities to avoid frequent interruptions for the woman. 2. Mothers should be counseled to sleep when the baby sleeps. 3. Mothers should be counseled to delegate or postpone unnecessary chores and activities and to sleep when the baby sleeps. 5. Mothers should be counseled to utilize family and friends for support.

During a visit to the obstetrician, a pregnant client questions the nurse about the potential need for an amniotomy. The nurse explains that an amniotomy is performed to do which of the following? Note: Credit will be given only if all correct choices and no incorrect choices are selected. Select all that apply. 1. Stimulate the beginning of labor 2. Augment labor progression 3. Allow application of an internal fetal electrode 4. Allow application of an external fetal monitor 5. Allow insertion of an intrauterine pressure catheter

Answer: 1, 2, 3, 5 Explanation: 1. Amniotomy is the artificial rupture of the amniotic membranes and can be used to induce labor. 2. Amniotomy can be done to augment labor. 3. Amniotomy allows access to the fetus in order to apply an internal fetal electrode to the fetal scalp. 5. Amniotomy may be performed during labor to allow an intrauterine pressure catheter to be inserted.

Childbirth preparation offers several advantages including which of the following? Note: Credit will be given only for all correct choices and no incorrect choices. Select all that apply. 1. It helps a pregnant woman and her support person understand the choices in the birth setting. 2. It promotes awareness of available options. 3. It provides tools for a pregnant woman and her support person to use during labor and birth. 4. Women who receive continuous support during labor require more analgesia, and have more cesarean and instrument births. 5. Each method has been shown to shorten labor.

Answer: 1, 2, 3, 5 Explanation: 1. Childbirth preparation offers several advantages. It helps a pregnant woman and her support person understand the choices in the birth setting, promotes awareness of available options, and provides tools for them to use during labor and birth. 2. Childbirth preparation offers several advantages. It helps a pregnant woman and her support person understand the choices in the birth setting, promotes awareness of available options, and provides tools for them to use during labor and birth. 3. Childbirth preparation offers several advantages. It helps a pregnant woman and her support person understand the choices in the birth setting, promotes awareness of available options, and provides tools for them to use during labor and birth. 5. Childbirth preparation offers several advantages. Each method has been shown to shorten labor.

The nurse is teaching a prenatal class about postpartum changes. The nurse explains that factors that might interfere with uterine involution include which of the following? Note: Credit will be given only if all correct choices and no incorrect choices are selected. Select all that apply. 1. Prolonged labor 2. Difficult birth 3. Full bladder 4. Breastfeeding 5. Infection

Answer: 1, 2, 3, 5 Explanation: 1. During prolonged labor, the muscles relax because of prolonged time of contraction during labor. 2. During a difficult birth, the uterus is manipulated excessively, causing fatigue. 3. As the uterus is pushed up and usually to the right, pressure on a full bladder interferes with effective uterine contraction. 5. Inflammation and infection interfere with uterine muscle's ability to contract effectively.

Upon assessing the FHR tracing, the nurse determines that there is fetal tachycardia. The fetal tachycardia would be caused by which of the following? Note: Credit will be given only if all correct choices and no incorrect choices are selected. Select all that apply. 1. Early fetalhypoxia 2. Prolonged fetal stimulation 3. Fetal anemia 4. Fetal sleep cycle 5. Infection

Answer: 1, 2, 3, 5 Explanation: 1. Early fetal hypoxia can cause fetal tachycardia. 2. Prolonged fetal stimulation can cause fetal tachycardia. 3. Fetal anemia can cause fetal tachycardia. 5. Infection can cause fetal tachycardia.

) In caring for a client with a uterine rupture, the nurse determines which nursing diagnoses to be appropriate? Note: Credit will be given only if all correct choices and no incorrect choices are selected. Select all that apply. 1. Gas Exchange, Impaired 2. Fear related to unknown outcome 3. Coping, Ineffective 4. Mobility: Physical, Impaired 5. Anxiety

Answer: 1, 2, 3, 5 Explanation: 1. Gas Exchange, Impaired diagnosis could apply to both mother and fetus. 2. The client would experience fear related to an unknown outcome. 3. Ineffective coping would be due to emergent situation secondary to uterine rupture. 5. There will be anxiety related to emergency procedures and unknown fetal outcome.

The nurse is teaching a class on reading a fetal monitor to nursing students. The nurse explains that bradycardia is a fetal heart rate baseline below 110 and can be caused by which of the following? Note: Credit will be given only if all correct choices and no incorrect choices are selected. Select all that apply. 1. Maternal hypotension 2. Prolonged umbilical cord compression 3. Fetal dysrhythmia 4. Central nervous system malformation 5. Late fetal asphyxia

Answer: 1, 2, 3, 5 Explanation: 1. Maternal hypotension results in decreased blood flow to the fetus. 2. Cord compression can cause fetal bradycardia. 3. This will cause fetal bradycardia if there is a fetal heart block. 5. This is a depression of myocardial activity.

At her 6-week postpartum checkup, a new mother voices concerns to the nurse. She states that she is finding it hard to have time alone to even talk on the phone without interruption. Her family lives in another state, and she has contact with them only by phone. She is still having difficulty getting enough sleep and worries that she will not be a good mother. Appropriate nursing interventions would include providing which of the following? Note: Credit will be given only if all correct choices and no incorrect choices are selected. Select all that apply. 1. Anticipatory guidance about the realities of being a parent. 2. Parenting literature and reference manuals. 3. Phone numbers and locations of local parenting groups. 4. Referral for specialized interventions related to postpartum blues. 5. Phone numbers and names of postpartum doulas.

Answer: 1, 2, 3, 5 Explanation: 1. Postpartum nurses need to be aware of the long-term adjustments and stresses that the childbearing family faces as its members adjust to new and different roles. 2. Postpartum nurses need to be aware of the long-term adjustments and stresses that the childbearing family faces as its members adjust to new and different roles. 3. New mother support groups are helpful for women who lack a social support system. 5. Postpartum doulas are professionals trained to help the new mother after the birth of the baby.

) The nurse has completed the physical assessment of a client in early labor, and proceeds with the social assessment. A social history of the client would include which of the following? Note: Credit will be given only if all correct choices and no incorrect choices are selected. Select all that apply. 1. Use of drugs and alcohol 2. Family violence or sexual assault 3. Current living situation 4. Type of insurance 5. Availability of resources

Answer: 1, 2, 3, 5 Explanation: 1. Risk factors such as the use of drugs or alcohol during the pregnancy can influence the labor and birth. 2. It is imperative to ask the woman about domestic violence and to assess any degree of psychologic or physical harm, either potential or real. 3. A social assessment includes asking about the woman's current living situation. This dialog provides an opportunity for the nurse to continue to build support, to provide information when requested, and to be direct yet supportive. 5. A social assessment includes asking about resources available to the family.

A prenatal client asks the nurse about conditions that would necessitate a cesarean delivery. The nurse explains that cesarean delivery generally is performed in the presence of which of the following? Note: Credit will be given only if all correct choices and no incorrect choices are selected. Select all that apply. 1. Complete placenta previa 2. Placental abruption 3. Umbilical cord prolapse 4. Precipitous labor 5. Failure to progress

Answer: 1, 2, 3, 5 Explanation: 1. When the placenta completely covers the uterine opening, a cesarean is performed. 2. Premature separation of the placenta from the uterine wall requires an immediate cesarean. 3. A prolapsed cord is an emergency requiring an immediate cesarean. 5. Failure to progress in labor can necessitate a cesarean birth.

Premonitory signs of labor include which of the following? Note: Credit will be given only for all correct choices and no incorrect choices. Select all that apply. 1. Braxton Hicks contractions 2. Cervical softening and effacement 3. Weight gain 4. Rupture of membranes 5. Sudden loss of energy

Answer: 1, 2, 4 Explanation: 1. A premonitory sign of labor includes Braxton Hicks contractions. 2. A premonitory sign of labor includes cervical softening and effacement. 4. A premonitory sign of labor includes rupture of membranes.

The nurse is caring for a client in labor. Which signs and symptoms would indicate the client is progressing into the second stage of labor? Note: Credit will be given only for all correct choices and no incorrect choices. Select all that apply. 1. Bulging perineum 2. Increased bloody show 3. Spontaneous rupture of the membranes 4. Uncontrollable urge to push 5. Inability to breathe through contractions

Answer: 1, 2, 4 Explanation: 1. As the fetal head continues its descent, the perineum begins to bulge, flatten, and move anteriorly. 2. Bloody show increases as a woman enters the second stage of labor. 4. As the fetal head descends, the woman has the urge to push because of pressure of the fetal head on the sacral and obturator nerves.

Risk factors for tachysystole include which of the following? Note: Credit will be given only if all correct choices and no incorrect choices are selected. Select all that apply. 1. Cocaine use 2. Placental abruption 3. Low-dose oxytocin titration regimens 4. Uterine rupture 5. Smoking

Answer: 1, 2, 4 Explanation: 1. Cocaine use is a risk factor for tachysystole. 2. Placental abruption is a risk factor for tachysystole. 4. Uterine rupture is a risk factor for tachysystole.

The client is undergoing an emergency cesarean birth for fetal bradycardia. The client's partner has not been allowed into the operating room. What can the nurse do to alleviate the partner's emotional distress? Note: Credit will be given only if all correct choices and no incorrect choices are selected. Select all that apply. 1. Allow the partner to wheel the baby's crib to the newborn nursery. 2. Allow the partner to be near the operating room where the newborn's first cry can be heard. 3. Have the partner wait in the client's postpartum room. 4. Encourage the partner to be in the nursery for the initial assessment. 5. Teach the partner how to take the client's blood pressure.

Answer: 1, 2, 4 Explanation: 1. Effective measures include allowing the partner to take the baby to the nursery. 2. Effective measures include allowing the partner to be in a place near the operating room, where the newborn's first cry can be heard. 4. Effective measures include involving the partner in postpartum care in the recovery room.

The nurse knows that the Bishop scoring system for cervical readiness includes which of the following? Note: Credit will be given only if all correct choices and no incorrect choices are selected. Select all that apply. 1. Fetal station 2. Fetal lie 3. Fetal presenting part 4. Cervical effacement 5. Cervical softness

Answer: 1, 4, 5 Explanation: 1. Fetal station is one of the components evaluated by the Bishop scoring system. 4. Cervical effacement is one of the components evaluated by the Bishop scoring system. 5. Cervical consistency is one of the components evaluated by the Bishop scoring system.

A full-term infant has just been born. Which interventions should the nurse perform first? Note: Credit will be given only if all correct choices and no incorrect choices are selected. Select all that apply. 1. Placing the infant in a radiant-heated unit 2. Suctioning the infant with a bulb syringe 3. Wrapping the infant in a blanket 4. Evaluating the newborn using the Apgar system 5. Offering a feeding of 5% glucose water

Answer: 1, 2, 4 Explanation: 1. If the newborn is placed in a radiant-heated unit, he or she is dried, laid on a dry blanket, and left uncovered under the radiant heat. 2. Newborns are suctioned with a bulb syringe to clear mucus from the newborn's mouth. 4. The purpose of the Apgar score is to evaluate the physical condition of the newborn at birth.

Which strategies would the nurse utilize to promote culturally competent care for the postpartum client? Note: Credit will be given only if all correct choices and no incorrect choices are selected. Select all that apply. 1. Examine one's own cultural beliefs, biases, stereotypes, and prejudices. 2. Respect the values and beliefs of others. 3. Limit the alternative food choices offered clients to minimize conflicts. 4. Incorporate the family's cultural practices into the care. 5. Evaluate whether the family's cultural practices fit into Western norms.

Answer: 1, 2, 4 Explanation: 1. It is important for nurses to recognize that they are approaching their patient's care from their own perspective and that, to individualize care for each mother, they need to assess the woman's preferences, her level of acculturation and assimilation to Western culture, her linguistic abilities, and her educational level. 2. It is important for nurses to recognize that they are approaching their patient's care from their own perspective and that, to individualize care for each mother, they need to assess the woman's preferences, her level of acculturation and assimilation to Western culture, her linguistic abilities, and her educational level. 4. The nurse should have the mother exercise her choices when possible and support those choices, with the help of cultural awareness and a sound knowledge base.

What maternity unit policies promote postpartal family wellness and shared parenting? Note: Credit will be given only if all correct choices and no incorrect choices are selected. Select all that apply. 1. Mother-baby care or couplet care on the postpartum unit 2. Skin-to-skin contact between the mother and baby and the father and baby 3. Newborn kept in the nursery to allow mother to rest between feedings 4. On-demand feeding schedule for both breastfed and bottle-fed infants 5. Limited visiting hours for the father so that the mother can sleep as needed

Answer: 1, 2, 4 Explanation: 1. Mother-baby care is an important part of the family-centered care approach. 2. The nurse should encourage skin-to-skin contact between mother (or father) and baby to promote breastfeeding and bonding. 4. The mother-baby unit is conducive to an on-demand feeding schedule for both breastfeeding and formula-feeding infants.

The nurse is caring for a client who is showing a sinusoidal fetal heart rate pattern on the monitor. The nurse knows that possible causes for this pattern include which of the following? Note: Credit will be given only if all correct choices and no incorrect choices are selected. Select all that apply. 1. Fetal anemia 2. Chronic fetal bleeding 3. Maternal hypotension 4. Twin-to-twin transfusion 5. Umbilical cord occlusion

Answer: 1, 2, 4, 5 Explanation: 1. Fetal anemia can cause a sinusoidal heart rate. 2. Chronic fetal bleeding can cause a sinusoidal heart rate. 4. Twin-to-twin transfusion will cause a sinusoidal heart rate. 5. Umbilical cord occlusion can cause a sinusoidal fetal heart rate.

The nurse is assessing a client before administering an analgesic. What are some of the factors the nurse should consider? Note: Credit will be given only if all correct choices and no incorrect choices are selected. Select all that apply. 1. The client is willing to receive medication after being advised about it. 2. The client's vital signs are stable. 3. The partner agrees to use of the medication. 4. The client has no known allergies to the medication. 5. The client is aware of the contraindications of the medication.

Answer: 1, 2, 4, 5 Explanation: 1. Medication should be explained to the client before it is administered. 2. Vital signs need to be stable before any analgesic medication is administered. 4. Ask the client about allergies before administering any medications. 5. Clients should always be aware of the contraindications of the medication.

When preparing for and performing an assessment of the postpartum client, which of the following would the nurse do? Note: Credit will be given only if all correct choices and no incorrect choices are selected. Select all that apply. 1. Ask the client to void before assessing the uterus. 2. Inform the client of the need for regular assessments. 3. Defer client teaching to another time. 4. Perform the procedures as gently as possible. 5. Take precautions to prevent exposure to body fluids.

Answer: 1, 2, 4, 5 Explanation: 1. Palpating the fundus when the woman has a full bladder may give false information about the progress of involution. Ask the woman to void before assessment. 2. The nurse should provide an explanation of the purposes of regular assessment to the woman. 4. The woman should be relaxed before starting, and procedures should be performed as gently as possible, to avoid unnecessary discomfort. 5. Gloves should be worn before starting the assessment.

The nurse is providing postpartum care to an obese client. As part of care for this client, the nurse should do which of the following? Note: Credit will be given only if all correct choices and no incorrect choices are selected. Select all that apply. 1. Apply sequential compression devices 2. Have the mother ambulate as early as possible 3. Encourage bottle-feeding over breastfeeding 4. Supervise breastfeeding 5. Instruct the client on signs of infection

Answer: 1, 2, 4, 5 Explanation: 1. The use of sequential compression devices (SCDs) and early ambulation are essential to the prevention of deep vein thrombosis, especially if the client had a cesarean birth. 2. Ambulation should be encouraged as soon as possible to prevent pneumonia. 4. The new mother may need extra supervision and assistance when breastfeeding her baby to ensure newborn safety. 5. The obese client has needs similar to all postpartum client, but she needs special attention to prevent injury, respiratory complications, thromboembolic disease, and infection, for which she is at high risk.

) The incidence of complications and discomforts in the first year postpartum is common and women may experience which of the following? Note: Credit will be given only if all correct choices and no incorrect choices are selected. Select all that apply. 1. Pain 2. Excess energy 3. Urinary incontinence 4. Changes in mental health status 5. Sleep deprivation

Answer: 1, 3, 4, 5 Explanation: 1. Pain can be a discomfort in the first year postpartum. 3. Urinary incontinence can be a complication in the first year postpartum. 4. Changes in mental health status can be a complication in the first year postpartum. 5. Sleep deprivation can be a complication in the first year postpartum.

When caring for a 13-year-old client in labor, how would the nurse provide sensitive care? Note: Credit will be given only if all correct choices and no incorrect choices are selected. Select all that apply. 1. Using simple and concrete instructions 2. Providing soothing encouragement and comfort measures 3. Making all decisions for the client when she expresses a feeling of helplessness 4. Deciding whom the client should allow in the room 5. Providing encouragement and support of the client's decisions

Answer: 1, 2, 5 Explanation: 1. A client at this developmental stage will need concrete and simplified instructions. 2. Touch, soothing encouragement, and measures to promote her comfort help her maintain control and meet her needs for dependence. 5. Establishing rapport without recrimination will provide emotional support and encouragement.

What information should the nurse include when teaching the postpartal client and partner about resumption of sexual activity? Note: Credit will be given only if all correct choices and no incorrect choices are selected. Select all that apply. 1. Couples should be encouraged to abstain from intercourse until the episiotomy is healed and the lochial flow has stopped. 2. Postpartum women often experience vaginal dryness, and should be encouraged to use some kind of lubrication initially during intercourse. 3. Breastfeeding the newborn after intercourse can reduce the chance of milk spouting from the nipples. 4. Maternal changes in libido are usually indicative of psychological depression. 5. Maternal fatigue is often a significant factor limiting the resumption of sexual intercourse.

Answer: 1, 2, 5 Explanation: 1. Currently, the couple is advised to abstain from intercourse until the episiotomy is healed and the lochia has stopped. 2. Because postpartum women often experience vaginal dryness due to hormonal changes, the use of a water-based lubrication, such as K-Y jelly or Astroglide, may initially be necessary during intercourse. 5. Maternal fatigue is often a significant factor limiting the resumption of sexual intercourse. While interest and desire vary, most couples resume sexual activity within 3 months.

The client presents to the labor and delivery unit stating that her water broke 2 hours ago. Indicators of normal labor include which of the following? Note: Credit will be given only if all correct choices and no incorrect choices are selected. Select all that apply. 1. Fetal heart rate of 130 with average variability 2. Blood pressure of 130/80 3. Maternal pulse of 160 4. Protein of +1 in urine 5. Odorless, clear fluid on underwear

Answer: 1, 2, 5 Explanation: 1. Fetal heart rate (FHR) of 110-160 with average variability is a normal indication. 2. Maternal vital sign of blood pressure below 140/90 is a normal indication. 5. Fluid clear and without odor if membranes ruptured is a normal indication.

Fetal factors that possibly indicate electronic fetal monitoring include which of the following? 1. Meconium passage 2. Multiple gestation 3. Preeclampsia 4. Grand multiparity 5. Decreased fetal movement

Answer: 1, 2, 5 Explanation: 1. Meconium passage is an indicator for electronic fetal monitoring. 2. Multiple gestation is an indicator for electronic fetal monitoring. 5. Decreased fetal movement is an indicator for electronic fetal monitoring.

3) A client in labor is requesting pain medication. The nurse assesses her labor status first, focusing on which of the following? Note: Credit will be given only if all correct choices and no incorrect choices are selected. Select all that apply. 1. Contraction pattern 2. Amount of cervical dilatation 3. When the labor began 4. Whether the membranes are intact or ruptured 5. Fetal presenting part

Answer: 1, 2, 5 Explanation: 1. The client should have a good contraction pattern before receiving an analgesic. 2. The nurse should evaluate the amount of cervical dilatation before analgesic medication is administered. 5. If normal parameters are absent or if nonreassuring maternal or fetal factors are present, the nurse may need to complete further assessments with the physician/CNM.

During labor, the client at 4 cm suddenly becomes short of breath, cyanotic, and hypoxic. The nurse must prepare or arrange immediately for which of the following? Note: Credit will be given only if all correct choices and no incorrect choices are selected. Select all that apply. 1. Intravenous access 2. Cesarean delivery 3. Immediate vaginal delivery 4. McRoberts maneuver 5. A crash cart

Answer: 1, 2, 5 Explanation: 1. When an amniotic fluid embolism is suspected, intravenous access is obtained as quickly as possible. 2. Shortness of breath, cyanosis, and hypoxia are symptoms of an amniotic fluid embolus, which necessitates immediate cesarean delivery. 5. The chances of a code are high, so the crash cart needs to be available.

A client had an epidural inserted 2 hours ago. It is functioning well, the client is stable, and labor is progressing. Which parts of the nurse's assessment have the highest priority? Note: Credit will be given only if all correct choices and no incorrect choices are selected. Select all that apply. 1. Assess blood pressure every hour. 2. Assess the pulse rate every hour. 3. Palpate the bladder. 4. Auscultate the lungs. 5. Assess the reflexes.

Answer: 1, 3 Explanation: 1. Blood pressure should be monitored every 1 to 2 minutes for the first 10 minutes and then every 5 to 15 minutes until the block wears off because hypotension is the most common side effect of epidural anesthesia. 3. Nursing care following an epidural block includes frequent assessment of the bladder to avoid bladder distention. Catheterization may be necessary, because most women are unable to void.

What are the primary complications of placenta accrete? Note: Credit will be given only if all correct choices and no incorrect choices are selected. Select all that apply. 1. Maternal hemorrhage 2. Insomnia 3. Failure of the placenta to separate following birth of the infant 4. Autonomic dysreflexia 5. Shoulder dystocia

Answer: 1, 3 Explanation: 1. The primary complications of placenta accreta are maternal hemorrhage and failure of the placenta to separate following birth of the infant. 3. The primary complications of placenta accreta are maternal hemorrhage and failure of the placenta to separate following birth of the infant.

Before performing Leopold maneuvers, what would the nurse do? Note: Credit will be given only if all correct choices and no incorrect choices are selected. Select all that apply. 1. Have the client empty her bladder. 2. Place the client in Trendelenburg position. 3. Have the client lie on her back with her feet on the bed and knees bent. 4. Turn the client to her left side. 5. Have the client lie flat with her ankles crossed.

Answer: 1, 3 Explanation: 1. The woman should have recently emptied her bladder before performing Leopold maneuvers. 3. The woman should lie on her back with her abdomen uncovered. To aid in relaxation of the abdominal wall, the shoulders should be raised slightly on a pillow and the knees drawn up a little.

When general anesthesia is necessary for a cesarean delivery, what should the nurse be prepared to do? Note: Credit will be given only if all correct choices and no incorrect choices are selected. Select all that apply. 1. Administer an antacid to the client. 2. Place a wedge under her thigh. 3. Apply cricoid pressure during anesthesia intubation. 4. Preoxygenate for 3-5 minutes before anesthesia. 5. Place a Foley catheter in the client's bladder.

Answer: 1, 3, 4, 5 Explanation: 1. Prophylactic antacid therapy is given to reduce the acidic content of the stomach before general anesthesia. 3. During the process of rapid induction of anesthesia, the nurse applies cricoid pressure. 4. The woman should be preoxygenated with 3 to 5 minutes of 100% oxygen. 5. Urinary retention can be treated with the placement of an indwelling Foley catheter.

Usually, the family is advised to arrive at the birth setting at the beginning of the active phase of labor or when which of the following occur? Note: Credit will be given only if all correct and no incorrect choices are selected. Select all that apply. 1. Rupture of membranes (ROM) 2. Increased fetal movement 3. Decreased fetal movement 4. Any vaginal bleeding 5. Regular, frequent uterine contractions (UCs)

Answer: 1, 3, 4, 5 Explanation: 1. The family is advised to arrive at the birth setting at the beginning of the active phase of labor or when the membranes rupture. 3. The family is advised to arrive at the birth setting at the beginning of the active phase of labor or when there is decreased fetal movement. 4. The family is advised to arrive at the birth setting at the beginning of the active phase of labor or when there is any vaginal bleeding. 5. The family is advised to arrive at the birth setting at the beginning of the active phase of labor or when there are regular, frequent uterine contractions.

The nurse is caring for a client who had a cesarean birth 4 hours ago. Which interventions would the nurse implement at this time? Note: Credit will be given only if all correct and no incorrect choices are selected. Select all that apply. 1. Administer analgesics as needed. 2. Encourage the client to ambulate to the bathroom to void. 3. Encourage leg exercises every 2 hours. 4. Encourage the client to cough and deep-breathe every 2 to 4 hours. 5. Encourage the use of breathing, relaxation, and distraction.

Answer: 1, 3, 4, 5 Explanation: 1. The nurse continues to assess the woman's pain level and provide relief measures as needed. 3. Within the first 12 hours postoperatively, unless medically contraindicated, the woman should be assisted to dangle her legs on the side of the bed. 4. The woman is encouraged to cough and breathe deeply and to use incentive spirometry every 2 to 4 hours while awake for the first few days following cesarean birth. 5. The nurse should encourage the use of breathing, relaxation, and distraction techniques.

A first-time 22-year-old single labor client, accompanied by her boyfriend, is admitted to the labor unit with ruptured membranes and mild to moderate contractions. She is determined to be 2 centimeters dilated. Which nursing diagnoses might apply during the current stage of labor? Note: Credit will be given only if all correct choices and no incorrect choices are selected. Select all that apply. 1. Fear/Anxiety related to discomfort of labor and unknown labor outcome 2. Knowledge, Deficient, related to lack of information about pushing methods 3. Pain, Acute, related to uterine contractions, cervical dilatation, and fetal descent 4. Pain, Acute, related to perineal trauma 5. Coping: Family, Compromised, related to labor process

Answer: 1, 3, 5 Explanation: 1. A Fear/Anxiety diagnosis would apply to the first stage of labor for a first-time labor client. 3. Contractions become more regular in frequency and duration, increasing discomfort and pain. 5. The woman and her boyfriend are about to undergo one of the most meaningful and stressful events in life together. Physical and psychologic resources, coping mechanisms, and support systems will all be challenged.

A clinic nurse is preparing diagrams of pelvic shapes. Which pelvic shapes are considered least adequate for vaginal childbirth? Note: Credit will be given only for all correct choices and no incorrect choices. Select all that apply. 1. Android 2. Anthropoid 3. Gynecoid 4. Platypelloid 5. Lambdoidal suture

Answer: 1, 4 Explanation: 1. In the android and platypelloid types, the pelvic diameters are diminished. Labor is more likely to be difficult (longer) and a cesarean birth is more likely. 4. In the android and platypelloid types, the pelvic diameters are diminished. Labor is more likely to be difficult (longer) and a cesarean birth is more likely.

The nurse is teaching a prenatal class about false labor. The nurse should teach clients that false labor most likely will include which of the following? Note: Credit will be given only for all correct choices and no incorrect choices. Select all that apply. 1. Contractions that do not intensify while walking 2. An increase in the intensity and frequency of contractions 3. Progressive cervical effacement and dilatation 4. Pain in the abdomen that does not radiate 5. Contractions are at regular intervals

Answer: 1, 4 Explanation: 1. True labor contractions intensify while walking. 4. The discomfort of true labor contractions usually starts in the back and radiates around to the abdomen.

The nurse is caring for a postpartum client who is experiencing afterpains following the birth of her third child. Which comfort measure should the nurse implement to decrease her pain? Note: Credit will be given only if all correct choices and no incorrect choices are selected. Select all that apply. 1. Offer a warm water bottle for her abdomen. 2. Call the physician to report this finding. 3. Inform her that this is not normal, and she will need an oxytocic agent. 4. Administer a mild analgesic to help with breastfeeding. 5. Administer a mild analgesic at bedtime to ensure rest.

Answer: 1, 4, 5 Explanation: 1. A warm water bottle placed against the low abdomen may reduce the discomfort of afterpains. 4. The breastfeeding mother may find it helpful to take a mild analgesic agent approximately 1 hour before feeding her infant. 5. An analgesic agent such as ibuprofen is also helpful at bedtime if the afterpains interfere with the mother's rest.

Which factors would the nurse observe that would indicate a new mother's early attachment to the newborn? Note: Credit will be given only if all correct choices and no incorrect choices are selected. Select all that apply. 1. Face-to-face contact and eye contact 2. Failure to choose a name for the baby 3. Decreased interest in the infant's cues 4. Pointing out familial traits of the newborn 5. Displaying satisfaction with the infant's sex

Answer: 1, 4, 5 Explanation: 1. Face-to-face contact and eye contact indicates that the mother is attracted to the infant and is attending to the infant's behavior. 4. The ability to point out family traits shows that she is pleased with the baby's appearance and recognizes the infant as belonging to the family unit. 5. Showing pleasure with the infant's appearance and sex indicates bonding is occurring.

A client who is having false labor most likely would have which of the following? Note: Credit will be given only for all correct choices and no incorrect choices. Select all that apply. 1. Contractions that do not intensify while walking 2. An increase in the intensity and frequency of contractions 3. Progressive cervical effacement and dilatation 4. Pain in the abdomen that does not radiate 5. Contractions that lessen with rest and warm tub baths

Answer: 1, 4, 5 Explanation: 1. True labor contractions intensify while walking. 4. True labor results in progressive dilation, increased intensity and frequency of contractions, and pain in the back that radiates to the abdomen. 5. In true labor, contractions do not lessen with rest and warm tub baths.

The nurse is caring for a client in the transition phase of labor and notes that the fetal monitor tracing shows average short-term and long-term variability with a baseline of 142 beats per minute. What actions should the nurse take in this situation? Note: Credit will be given only if all correct choices and no incorrect choices are selected. Select all that apply. 1. Provide caring labor support. 2. Administer oxygen via face mask. 3. Change the client's position. 4. Speed up the client's intravenous. 5. Reassure the client and her partner that she is doing fine.

Answer: 1, 5 Explanation: 1. The tracing is normal, so the nurse can continue support of the labor. 5. The nurse can reassure the client at this time, as the tracing is normal.

The nurse expects an initial weight loss for the average postpartum client to be which of the following? 1. 5 to 8 pounds 2. 10 to 12 pounds 3. 12 to 15 pounds 4. 15 to 20 pounds

Answer: 2 Explanation: 2. An initial weight loss of 10 to 12 lbs. occurs as a result of the birth of infant, placenta, and amniotic fluid.

Narcotic analgesia is administered to a laboring client at 10:00 a.m. The infant is delivered at 12:30 p.m. What would the nurse anticipate that the narcotic analgesia could do? 1. Be used in place of preoperative sedation 2. Result in neonatal respiratory depression 3. Prevent the need for anesthesia with an episiotomy 4. Enhance uterine contractions

Answer: 2 Explanation: 2. Analgesia given too late is of no value to the woman and may cause neonatal respiratory depression.

During labor, the fetus was in a brow presentation, but after a prolonged labor, the fetus converted to face presentation and was delivered vaginally with forceps assist. What should the nurse explain to the parents? 1. The infant will need to be observed for meconium aspiration. 2. Facial edema and head molding will subside in a few days. 3. The infant will be given prophylactic antibiotics. 4. Breastfeeding will need to be delayed for a day or two.

Answer: 2 Explanation: 2. Any facial edema and head molding that result from the use of forceps at birth will subside in a few days.

Upon delivery of the newborn, what nursing intervention most promotes parental attachment? 1. Placing the newborn under the radiant warmer. 2. Placing the newborn on the mother's abdomen. 3. Allowing the mother a chance to rest immediately after delivery. 4. Taking the newborn to the nursery for the initial assessment.

Answer: 2 Explanation: 2. As the baby is placed on the mother's abdomen or chest, she frequently reaches out to touch and stroke her baby. When the newborn is placed in this position, the father or partner also has a very clear, close view and can also reach out to touch the baby.

The labor and delivery nurse is reviewing charts. The nurse should inform the supervisor about which client? 1. Client at 5 cm requesting labor epidural analgesia 2. Client whose cervix remains at 6 cm for 4 hours 3. Client who has developed nausea and vomiting 4. Client requesting her partner to stay with her

Answer: 2 Explanation: 2. Average cervical change in the active phase of the first stage of labor is 1.2 cm/hour; thus, this client's lack of cervical change is unexpected, and should be reported to the supervisor.

A woman in labor asks the nurse to explain the electronic fetal heart rate monitor strip. The fetal heart rate baseline is 150 with accelerations to 165, variable decelerations to 140, and moderate long-term variability. Which statement indicates that the client understands the nurse's teaching? 1. "The most important part of fetal heart monitoring is the absence of variable decelerations." 2. "The most important part of fetal heart monitoring is the presence of variability." 3. "The most important part of fetal heart monitoring is the fetal heart rate baseline." 4. "The most important part of fetal heart monitoring is the depth of decelerations."

Answer: 2 Explanation: 2. Baseline variability is a reliable indicator of fetal cardiac and neurologic function and well-being. The opposing "push-pull" balancing between the sympathetic nervous system and the parasympathetic nervous system directly affects the FHR.

The laboring client is having moderately strong contractions lasting 60 seconds every 3 minutes. The fetal head is presenting at a -2 station. The cervix is 6 cm and 100% effaced. The membranes spontaneously ruptured prior to admission, and clear fluid is leaking. Fetal heart tones are in the 140s with accelerations to 150. Which nursing action has the highest priority? 1. Encourage the husband to remain in the room. 2. Keep the client on bed rest at this time. 3. Apply an internal fetal scalp electrode. 4. Obtain a clean-catch urine specimen.

Answer: 2 Explanation: 2. Because the membranes are ruptured and the head is high in the pelvis at a -2 station, the client should be maintained on bed rest to prevent cord prolapse.

The laboring client participated in childbirth preparation classes that strongly discouraged the use of medications and intervention during labor. The client has been pushing for two hours, and is exhausted. The physician requests that a vacuum extractor be used to facilitate the birth. The client first states that she wants the birth to be normal, then allows the vacuum extraction. Following this, what should the nurse assess the client for after the birth? 1. Elation, euphoria, and talkativeness 2. A sense of failure and loss 3. Questions about whether or not to circumcise 4. Uncertainty surrounding the baby's name

Answer: 2 Explanation: 2. Clients who participate in childbirth classes that stress the normalcy of birth may feel a sense of loss or failure if an intervention is used during their labor or birth.

What is required for any women receiving oxytocin (Pitocin)? 1. CPR 2. Continuous electronic fetal monitoring 3. Administering oxygen by mask 4. Nonstress test

Answer: 2 Explanation: 2. Continuous electronic fetal monitoring (EFM) is required for any women receiving oxytocin (Pitocin).

The nurse is training a nurse new to the labor and delivery unit. They are caring for a laboring client who will have a forceps delivery. Which action or assessment finding requires intervention? 1. Regional anesthesia is administered via pudendal block. 2. The client is instructed to push between contractions. 3. Fetal heart tones are consistently between 110 and 115. 4. The client's bladder is emptied using a straight catheter.

Answer: 2 Explanation: 2. During the contraction, as the forceps are applied, the woman should avoid pushing.

A client is admitted to the labor unit with contractions 1-2 minutes apart lasting 60-90 seconds. The client is apprehensive and irritable. This client is most likely in what phase of labor? 1. Active 2. Transition 3. Latent 4. Second

Answer: 2 Explanation: 2. During transition, contractions have a frequency of 1 1/2 to 2 minutes, a duration of 60 to 90 seconds, and are strong in intensity. When the woman enters the transition phase, she may demonstrate significant anxiety.

The client at 39 weeks' gestation is undergoing a cesarean birth due to breech presentation. General anesthesia is being used. Which situation requires immediate intervention? 1. The baby's hands and feet are blue at 1 minute after birth. 2. The fetal heart rate is 70 prior to making the skin incision. 3. Clear fluid is obtained from the baby's oropharynx. 4. The neonate cries prior to delivery of the body.

Answer: 2 Explanation: 2. Fetal bradycardia occurs when the fetal heart rate falls below 110 beats/minute during a 10-minute period of continuous monitoring. When fetal bradycardia is accompanied by decreased variability, it is considered ominous and could be a sign of fetal compromise.

The physician has determined the need for forceps. The nurse should explain to the client that the use of forceps is indicated because of which of the following? 1. Her support person is exhausted 2. Premature placental separation 3. To shorten the first stage of labor 4. To prevent fetal distress

Answer: 2 Explanation: 2. Fetal conditions indicating the need for forceps include premature placental separation, prolapsed umbilical cord, and nonreassuring fetal status.

The client is in the second stage of labor. The fetal heart rate baseline is 170, with minimal variability present. The nurse performs fetal scalp stimulation. The client's partner asks why the nurse did that. What is the best response by the nurse? 1. "I stimulated the top of the fetus's head to wake him up a little." 2. "I stimulated the top of the fetus's head to try to get his heart rate to accelerate." 3. "I stimulated the top of the fetus's head to calm the fetus down before birth." 4. "I stimulated the top of the fetus's head to find out whether he is in distress."

Answer: 2 Explanation: 2. Fetal scalp stimulation is done when there is a question regarding fetal status. An acceleration indicates fetal well-being.

The client has undergone an ultrasound, which estimated fetal weight at 4500 g (9 pounds 14 ounces). Which statement indicates that additional teaching is needed? 1. "Because my baby is big, I am at risk for excessive bleeding after delivery." 2. "Because my baby is big, his blood sugars could be high after he is born." 3. "Because my baby is big, my perineum could experience trauma during the birth." 4. "Because my baby is big, his shoulders could get stuck and a collarbone broken."

Answer: 2 Explanation: 2. Hypoglycemia, not hyperglycemia, is a potential complication experienced by a macrosomic fetus.

The nursing instructor is conducting a class about attachment behaviors. Which statement by a student indicates the need for further instruction? 1. "The en face position promotes bonding and attachment." 2. "Ideally, initial skin-to-skin contact occurs after the baby has been assessed and bathed." 3. "In reciprocity, the interaction of mother and infant is mutually satisfying and synchronous." 4. "The needs of the mother and of her infant are balanced during the phase of mutual regulation."

Answer: 2 Explanation: 2. Ideally, initial skin-to-skin contact is immediate. The benefits of this practice are supported by a preponderance of evidence.

The physicians/CNM opts to use a vacuum extractor for a delivery. What does the nurse understand? 1. There is little risk with vacuum extraction devices. 2. There should be further fetal descent with the first two "pop-offs." 3. Traction is applied between contractions. 4. The woman often feels increased discomfort during the procedure.

Answer: 2 Explanation: 2. If more than three "pop-offs" occur (the suction cup pops off the fetal head), the procedure should be discontinued. Page Ref: 641

A new grandmother comments that when her children were born, they stayed in the nursery. The grandmother asks the nurse why her daughter's baby stays mostly in the room instead of the nursery. How should the nurse respond? 1. "Babies like to be with their mothers more than they like to be in the nursery." 2. "Contact between parents and babies increases attachment." 3. "Budget cuts have decreased the number of nurses in the nursery." 4. "Why do you ask? Do you have concerns about your daughter's parenting?"

Answer: 2 Explanation: 2. In a mother-baby unit, the newborn's crib is placed near the mother's bed, where she can see her baby easily; this is conducive to an on-demand feeding schedule for both breastfeeding and formula-feeding infants.

A client arrives in the labor and delivery unit and describes her contractions as occurring every 10-12 minutes, lasting 30 seconds. She is smiling and very excited about the possibility of being in labor. On exam, her cervix is dilated 2 cm, 100% effaced, and -2 station. What best describes this labor? 1. Second phase 2. Latent phase 3. Active phase 4. Transition phase

Answer: 2 Explanation: 2. In the early or latent phase of the first stage of labor, contractions are usually mild. The woman feels able to cope with the discomfort. The woman is often talkative and smiling and is eager to talk about herself and answer questions.

The laboring client and her partner have arrived at the birthing unit. Which step of the admission process should be undertaken first? 1. The sterile vaginal exam 2. Welcoming the couple 3. Auscultation of the fetal heart rate 4. Checking for ruptured membranes

Answer: 2 Explanation: 2. It is important to establish rapport and to create an environment in which the family feels free to ask questions. The support and encouragement of the nurse in maintaining a caring environment begin with the initial admission.

When comparing the anterior and posterior fontanelles of a newborn, the nurse knows that both are what? 1. Both are approximately the same size 2. Both close within 12 months of birth 3. Both are used in labor to identify station 4. Both allow for assessing the status of the newborn after birth

Answer: 4 Explanation: 4. The anterior and posterior fontanelles are clinically useful in identifying the position of the fetal head in the pelvis and in assessing the status of the newborn after birth.

A client calls the labor and delivery unit and tells the nurse that she is 39 weeks pregnant and that over the last 4 or 5 days, she has noticed that although her breathing has become easier, she is having leg cramps, a slight amount of edema in her lower legs, and an increased amount of vaginal secretions. The nurse tells the client that she has experienced which of the following? 1. Engagement 2. Lightening 3. Molding 4. Braxton Hicks contractions

Answer: 2 Explanation: 2. Lightening describes the effect occurring when the fetus begins to settle into the pelvic inlet.

The need for forceps has been determined. The client's cervix is dilated to 10 cm, and the fetus is at +2 station. What category of forceps application would the nurse anticipate? 1. Input 2. Low 3. Mid 4. Outlet

Answer: 2 Explanation: 2. Low forceps are applied when the leading edge of the fetal head is at +2 station.

The laboring client brought a written birth plan indicating that she wanted to avoid pain medications and an epidural. She is now at 6 cm and states, "I can't stand this anymore! I need something for pain! How will an epidural affect my baby?" What is the nurse's best response? 1. "The narcotic in the epidural will make both you and the baby sleepy." 2. "It is unlikely that an epidural will decrease your baby's heart rate." 3. "Epidurals tend to cause low blood pressure in babies after birth." 4. "I can't get you an epidural, because of your birth plan."

Answer: 2 Explanation: 2. Maternal hypotension results in uteroplacental insufficiency in the fetus, which is manifested as late decelerations on the fetal monitoring strip. The risk of hypotension can be minimized by hydrating the vascular system with 500 to 1000 mL of IV solution before the procedure and changing the woman's position and/or increasing the IV rate afterward.

A client received epidural anesthesia during the first stage of labor. The epidural is discontinued immediately after delivery. This client is at increased risk for which problem during the fourth stage of labor? 1. Nausea 2. Bladder distention 3. Uterine atony 4. Hypertension

Answer: 2 Explanation: 2. Nursing care following an epidural block includes frequent assessment of the bladder to avoid bladder distention.

The nurse should anticipate the labor pattern for a fetal occiput posterior position to be which of the following? 1. Shorter than average during the latent phase 2. Prolonged as regards the overall length of labor 3. Rapid during transition 4. Precipitous

Answer: 2 Explanation: 2. Occiput posterior (OP) position of the fetus is the most common fetal malposition and occurs when the head remains in the direct OP position throughout labor. This can prolong the overall length of labor.

What type of forceps are designed to be used with a breech presentation? 1. Midforceps 2. Piper 3. Low 4. High

Answer: 2 Explanation: 2. Piper forceps are designed to be used with a breech presentation. They are applied after the birth of the body, when the fetal head is still in the birth canal and assistance is needed.

The client delivered 30 minutes ago. Her blood pressure and pulse are stable. Vaginal bleeding is scant. The nurse should prepare for which procedure? 1. Abdominal hysterectomy 2. Manual removal of the placenta 3. Repair of perineal lacerations 4. Foley catheterization

Answer: 2 Explanation: 2. Retention of the placenta beyond 30 minutes after birth is termed retained placenta. Manual removal of the placenta is then performed.

Four minutes after the birth of a baby, there is a sudden gush of blood from the mother's vagina, and about 8 inches of umbilical cord slides out. What action should the nurse take first? 1. Place the client in McRoberts position. 2. Watch for the emergence of the placenta. 3. Prepare for the delivery of an undiagnosed twin. 4. Place the client in a supine position.

Answer: 2 Explanation: 2. Signs of placental separation usually appear around 5 minutes after birth of the infant, but can take up to 30 minutes to manifest. These signs are (1) a globular-shaped uterus, (2) a rise of the fundus in the abdomen, (3) a sudden gush or trickle of blood, and (4) further protrusion of the umbilical cord out of the vagina.

A woman has been admitted for an external version. She has completed an ultrasound exam and is attached to the fetal monitor. Prior to the procedure, why will terbutaline be administered? 1. To provide analgesia 2. To relax the uterus 3. To induce labor 4. To prevent hemorrhage

Answer: 2 Explanation: 2. Terbutaline is administered to achieve uterine relaxation.

The laboring client is at 7 cm, with the vertex at a +1 station. Her birth plan indicates that she and her partner took Lamaze prenatal classes, and they have planned on a natural, unmedicated birth. Her contractions are every 3 minutes and last 60 seconds. She has used relaxation and breathing techniques very successfully in her labor until the last 15 minutes. Now, during contractions, she is writhing on the bed and screaming. Her labor partner is rubbing the client's back and speaking to her quietly. Which nursing diagnosis should the nurse incorporate into the plan of care for this client? 1. Fear/Anxiety related to discomfort of labor and unknown labor outcome 2. Pain, Acute, related to uterine contractions, cervical dilatation, and fetal descent 3. Coping: Family, Compromised, related to labor process 4. Knowledge, Deficient, related to lack of information about normal labor process and comfort measures

Answer: 2 Explanation: 2. The client is exhibiting signs of acute pain, which is both common and expected in the transitional phase of labor.

Every time the nurse enters the room of a postpartum client who gave birth 3 hours ago, the client asks something else about her birth experience. What action should the nurse take? 1. Answer questions quickly and try to divert her attention to other subjects. 2. Review the documentation of the birth experience and discuss it with her. 3. Contact the physician to warn him the client might want to file a lawsuit, based on her preoccupation with the birth experience. 4. Submit a referral to Social Services because of possible obsessive behavior.

Answer: 2 Explanation: 2. The client may talk about her labor and birth experience. The nurse should provide opportunities to discuss the birth experience in a nonjudgmental atmosphere if the woman desires to do so.

A postpartum client has just received a rubella vaccination. The client demonstrates understanding of the teaching associated with administration of this vaccine when she states which of the following? 1. "I will need another vaccination in 3 months." 2. "I must avoid getting pregnant for 1 month." 3. "This will prevent me from getting chickenpox." 4. "This will protect my newborn from getting the measles."

Answer: 2 Explanation: 2. The client must avoid pregnancy for at least 1 month after receiving the rubella vaccine.

The client presents for cervical ripening in anticipation of labor induction tomorrow. What should the nurse include in her plan of care for this client? 1. Apply an internal fetal monitor. 2. Monitor the client using electronic fetal monitoring. 3. Withhold oral intake and start intravenous fluids. 4. Place the client in a upright, sitting position.

Answer: 2 Explanation: 2. The client should be monitored using electronic fetal monitoring for at least 30 minutes and up to 2 hours after placement to assess the contraction pattern and the fetal status.

The nurse is admitting a client to the birthing unit. What question should the nurse ask to gain a better understanding of the client's psychosocial status? 1. "How did you decide to have your baby at this hospital?" 2. "Who will be your labor support person?" 3. "Have you chosen names for your baby yet?" 4. "What feeding method will you use for your baby?"

Answer: 2 Explanation: 2. The expectant mother's partner or support person is an important member of the birthing team, and assessments of the couple's coping, interactions, and teamwork are integral to the nurse's knowledge base. The nurse's physical presence with the laboring woman provides the best opportunity for ongoing assessment.

In succenturiate placenta, one or more accessory lobes of fetal villi have developed on the placenta, with vascular connections of fetal origin. What is the gravest maternal danger? 1. Cord prolapse 2. Postpartum hemorrhage 3. Paroxysmal hypertension 4. Brachial plexus injury

Answer: 2 Explanation: 2. The gravest maternal danger is postpartum hemorrhage if this minor lobe is severed from the placenta and remains in the uterus.

A fetal weight is estimated at 4490 grams in a client at 38 weeks' gestation. Counseling should occur before labor regarding which of the following? 1. Mother's undiagnosed diabetes 2. Likelihood of a cesarean delivery 3. Effectiveness of epidural anesthesia with a large fetus 4. Need for early delivery

Answer: 2 Explanation: 2. The likelihood of a cesarean delivery with a fetus over 4000 grams is high. This should be discussed with the client before labor.

Dystocia encompasses many problems in labor. What is the most common? 1. Meconium-stained amniotic fluid 2. Dysfunctional uterine contractions 3. Cessation of contractions 4. Changes in the fetal heart rate

Answer: 2 Explanation: 2. The most common problem is dysfunctional (or uncoordinated) uterine contractions that result in a prolongation of labor.

Which statement by a nursing student preparing to care for a postpartum lesbian mother would indicate the need for further teaching? 1. "I can't let the client know I've never worked with lesbian mothers." 2. "I will have to adjust some of my discharge instruction for this mother." 3. "I don't need to include the partner when I provide care and instruction." 4. "Discharge teaching is exactly the same for lesbian mothers as for all others."

Answer: 2 Explanation: 2. The nurse should be aware that standardized postpartum instructions, particularly those related to intercourse and contraception might need to be individualized and amended.

A client is preparing to take a sitz bath for the first time. What will the nurse do? 1. Allow the client privacy during the sitz bath. 2. Place a call bell well within reach and check on the client frequently. 3. Discourage the client from taking a sitz bath. 4. Check on the client after the sitz bath.

Answer: 2 Explanation: 2. The nurse should explain the purpose and use of the sitz bath, anticipated effects, benefits, possible problems, and safety measures to prevent slipping or an injury from hot water. A call bell would be a safety measure.

The community nurse is meeting a new mother for the first time. The client delivered her first child 5 days ago after a 12-hour labor. Neither the mother nor the infant had any complications during the birth or postpartum period. Which statement by the client would indicate to the nurse that the client is experiencing postpartum blues? 1. "I am so happy and blessed to have my new baby." 2. "One minute I'm laughing and the next I'm crying." 3. "My husband is helping out by changing the baby at night." 4. "Breastfeeding is going quite well now that the engorgement is gone."

Answer: 2 Explanation: 2. The postpartum blues consist of a transient period of depression that occurs during the first few days of puerperium. Symptoms may include mood swings, anger, weepiness, anorexia, difficulty sleeping, and a feeling of letdown.

A laboring client has received an order for epidural anesthesia. In order to prevent the most common complication associated with this procedure, what would the nurse expect to do? 1. Observe fetal heart rate variability 2. Hydrate the vascular system with 500-1000 mL of intravenous fluids 3. Place the client in the semi-Fowler's position 4. Teach the client appropriate breathing techniques

Answer: 2 Explanation: 2. The risk of hypotension can be minimized by hydrating the vascular system with 500 to 1000 mL of IV solution before the procedure and changing the woman's position and/or increasing the IV rate afterward.

The neonate was born 5 minutes ago. The body is bluish. The heart rate is 150. The infant is crying strongly. The infant cries when the sole of the foot is stimulated. The arms and legs are flexed, and resist straightening. What should the nurse record as this infant's Apgar score? 1. 7 2. 8 3. 9 4. 10

Answer: 2 Explanation: 2. The strong cry earns 2 points. The crying with foot sole stimulation earns 2 points. The limb flexion and resistance earn 2 points each. Bluish color earns 0 points. The Apgar score is 8.

The client having her second child is scheduled for a cesarean birth because the baby is in a breech presentation. The client states, "I'm wondering what will be different this time compared with my first birth, which was vaginal." What response is best? 1. "We'll take good care of you and your baby. You'll be home before you know it." 2. "You'll be wearing a sequential compression device until you start walking." 3. "You will have a lot of pain, but there are medications that we give when it gets really bad." 4. "You won't be able to nurse until the baby is 12 hours old, because of your epidural."

Answer: 2 Explanation: 2. The use of sequential compression devices (SCDs) and early ambulation are essential to the prevention of deep vein thrombosis, especially if the client had a cesarean birth.

When caring for a new mother after cesarean birth, what complications would the nurse anticipate? Note: Credit will be given only if all correct choices and no incorrect choices are selected. Select all that apply. 1. Back pain 2. Pulmonary infection 3. Deep vein thrombosis 4. Pulmonary embolism 5. Perineal edema

Answer: 2, 3, 4 Explanation: 2. Immobility after delivery increases the risk of pulmonary infection. 3. Immobility after delivery increases the risk of deep vein thrombosis. 4. Immobility after delivery increases the risk of pulmonary embolism.

The nurse has received the end-of-shift report on the postpartum unit. Which client should the nurse see first? 1. Woman who is 2nd day post-cesarean, moderate lochia serosa 2. Woman day of delivery, fundus firm 2 cm above umbilicus 3. Woman who had a cesarean section, 1st postpartum day, 4 cm diastasis recti abdominis 4. Woman who had a cesarean section, 1st postpartum day, hypoactive bowel sounds all quadrants

Answer: 2 Explanation: 2. This client is the top priority. The fundus should not be positioned above the umbilicus after delivery. If the fundus is in the midline but higher than expected, it is usually associated with clots within the uterus.

The client at 40 weeks' gestation reports to the nurse that she has had increased pelvic pressure and increased urinary frequency. Which response by the nurse is best? 1. "Unless you have pain with urination, we don't need to worry about it." 2. "These symptoms usually mean the baby's head has descended further." 3. "Come in for an appointment today and we'll check everything out." 4. "This might indicate that the baby is no longer in a head-down position."

Answer: 2 Explanation: 2. This is the best response because it most directly addresses what the client has reported.

The nurse is working with a pregnant adolescent. The client asks the nurse how the baby's condition is determined during labor. The nurse's best response is that during labor, the nurse will do which of the following? 1. Check the client's cervix by doing a pelvic exam every 2 hours. 2. Assess the fetus's heart rate with an electronic fetal monitor. 3. Look at the color and amount of bloody show that the client has. 4. Verify that the client's contractions are strong but not too close together.

Answer: 2 Explanation: 2. This statement best answers the question the client has asked.

Before applying a cord clamp, the nurse assesses the umbilical cord. The mother asks why the nurse is doing this. What should the nurse reply? 1. "I'm checking the blood vessels in the cord to see whether it has one artery and one vein." 2. "I'm checking the blood vessels in the cord to see whether it has two arteries and one vein." 3. "I'm checking the blood vessels in the cord to see whether it has two veins and one artery." 4. "I'm checking the blood vessels in the cord to see whether it has two arteries and two veins."

Answer: 2 Explanation: 2. Two arteries and one vein are present in a normal umbilical cord.

The client has been pushing for two hours, and is exhausted. The fetal head is visible between contractions. The physician informs the client that a vacuum extractor could be used to facilitate the delivery. Which statement indicates that the client needs additional information about vacuum extraction assistance? 1. "A small cup will be put onto the baby's head, and a gentle suction will be applied." 2. "I can stop pushing and just rest if the vacuum extractor is used." 3. "The baby's head might have some swelling from the vacuum cup." 4. "The vacuum will be applied for a total of ten minutes or less."

Answer: 2 Explanation: 2. Vacuum extraction is an assistive delivery. The physician/CNM applies traction in coordination with uterine contractions.

The postpartum client expresses concern about getting back to her prepregnant shape, and asks the nurse when she will be able to run again. Which statement by the client indicates that teaching was effective? 1. "I can start running in 2 weeks, and can breastfeed as soon as I am done." 2. "I should see how my energy level is at home, and increase my activity slowly." 3. "Running is not recommended for breastfeeding women." 4. "If I am getting 8 hours of sleep per day, I can start running."

Answer: 2 Explanation: 2. Women should be encouraged to limit the number of activities to prevent excessive fatigue, increase in lochia, and negative psychologic reactions, such as feeling overwhelmed. A regular exercise program including vigorous activities such as running, weight lifting, or competitive sports can usually be initiated after the 6-week postpartum examination or when approved by the client's physician/CNM.

A multiparous client delivered her first child vaginally 2 years ago, and delivered an infant by cesarean yesterday due to breech presentation. Which statement would the nurse expect the client to make? 1. "I can't believe how much more tired I was with the first baby." 2. "I'm having significantly more pain this time than with my last birth." 3. "It is disappointing that I can't breastfeed because of the cesarean." 4. "Getting in and out of bed feels more comfortable than last time."

Answer: 2 Explanation: 2. Women with cesarean births have special needs: increased need for rest and sleep; incisional care; self-care; and relief of pain and discomfort.

A client has just arrived in the birthing unit. What steps would be most important for the nurse to perform to gain an understanding of the physical status of the client and her fetus? Note: Credit will be given only if all correct choices and no incorrect choices are selected. Select all that apply. 1. Check for ruptured membranes and apply a fetal scalp electrode. 2. Auscultate the fetal heart rate between and during contractions. 3. Palpate contractions and resting uterine tone. 4. Assess the blood pressure, temperature, respiratory rate, and pulse rate. 5. Perform a vaginal exam for cervical dilation, and perform Leopold maneuvers.

Answer: 2, 3 Explanation: 2. Fetal heart rate auscultation gives information about the physical status of the fetus. 3. Contraction palpation provides information about the frequency, duration, and intensity of the contractions.

The nurse is working with a new mother who follows Muslim traditions. Which expectations and actions are appropriate for this client? Note: Credit will be given only if all correct choices and no incorrect choices are selected. Select all that apply. 1. To be sure she gets a kosher diet. 2. Expect that most visitors will be women. 3. Uncover only the necessary skin when assessing. 4. The father will take an active role in infant care. 5. She will prefer a male physician.

Answer: 2, 3 Explanation: 2. In Muslim cultures, emphasis on childrearing and infant care activities is on the mother and female relatives. 3. Women of the Islamic faith may have specific modesty requirements; the woman must be completely covered, with only her feet and hands exposed.

Under which circumstances would the nurse remove prostaglandin from the client's cervix? Note: Credit will be given only if all correct choices and no incorrect choices are selected. Select all that apply. 1. Contractions every 5 minutes 2. Nausea and vomiting 3. Uterine tachysystole 4. Cardiac tachysystole 5. Baseline fetal heart rate of 140-148

Answer: 2, 3, 4 Explanation: 2. A reason to remove prostaglandin from a client's cervix is the presence of nausea and vomiting. 3. A reason to remove prostaglandin from a client's cervix is uterine tachysystole. 4. A reason to remove prostaglandin from a client's cervix is cardiac tachysystole.

The nurse is caring for a client who delivered by cesarean birth. The client received a general anesthetic. To prevent or minimize abdominal distention, which of the following would the nurse encourage? Note: Credit will be given if all correct choices and no incorrect choices are selected. Select all that apply. 1. Increased intake of cold beverages 2. Leg exercises every 2 hours 3. Abdominal tightening 4. Ambulation 5. Using a straw when drinking fluids

Answer: 2, 3, 4 Explanation: 2. Immobility increases the risk of abdominal distention and discomfort. Leg exercises serve to prevent or minimize abdominal distention in a surgical client who received a general anesthetic. 3. Abdominal tightening serves to prevent or minimize abdominal distention in a surgical client who received a general anesthetic. 4. Early ambulation prevents abdominal distention that can occur with excess accumulation of gas in the intestines.

The nurse knows that which of the following are advantages of spinal block? Note: Credit will be given only if all correct answers and no incorrect answers are selected. Select all that apply. 1. Intense blockade of sympathetic fibers 2. Relative ease of administration 3. Maternal compartmentalization of the drug 4. Immediate onset of anesthesia 5. Larger drug volume

Answer: 2, 3, 4 Explanation: 2. One of advantages of spinal block is the relative ease of administration. 3. One of the advantages of spinal block is the maternal compartmentalization of the drug. 4. One of the advantages of spinal block is the immediate onset of anesthesia.

Nonreassuring fetal status often occurs with a tachysystole contraction pattern. Intrauterine resuscitation measures may become warranted and can include which of the following measures? Note: Credit will be given only if all correct choices and no incorrect choices are selected. Select all that apply. 1. Position the woman on her right side. 2. Apply oxygen via face mask. 3. Call for anesthesia provider for support. 4. Increase intravenous fluids by at least 700 mL bolus. 5. Call the physician/CNM to the bedside.

Answer: 2, 3, 4 Explanation: 2. The nurse would apply oxygen via face mask. 3. The nurse would call for anesthesia provider for support. 4. The nurse would increase intravenous fluids by at least 500 mL bolus.

The nurse knows that the maternal risks associated with postterm pregnancy include which of the following? Note: Credit will be given only if all correct choices and no incorrect choices are selected. Select all that apply. 1. Polyhydramnios 2. Maternal hemorrhage 3. Maternal anxiety 4. Forceps-assisted delivery 5. Perineal damage

Answer: 2, 3, 4, 5 Explanation: 2. Maternal symptoms and complications in postterm pregnancy may include maternal hemorrhage. 3. Maternal symptoms and complications in postterm pregnancy may include maternal anxiety. 4. Maternal symptoms and complications in postterm pregnancy may include an operative vaginal birth with forceps or vacuum extractor. 5. Maternal symptoms and complications in postterm pregnancy may include perineal trauma and damage.

The nurse is caring for a client who plans to relinquish her baby for adoption. The nurse would implement which approach to care? Note: Credit will be given only if all correct choices and no incorrect choices are selected. Select all that apply. 1. Encourage the client to see and hold her infant. 2. Encourage the client to express her emotions. 3. Respect any special requests for the birth. 4. Acknowledge the grieving process in the client. 5. Allow access to the infant, if the client requests it.

Answer: 2, 3, 4, 5 Explanation: 2. The mother who decides to relinquish her baby needs emotional support and validation of her loss. 3. The woman should decide whether to see and hold her baby and should have any special requests regarding the birth honored. 4. Perinatal nurses should be aware that relinquishing mothers are at risk for disenfranchised grief, in which they are unable to proceed through the grieving process and come to resolution with the loss. The nurse should acknowledge the woman's loss and support her decision. 5. The amount of contact she chooses to have with her newborn should be respected.

Which of the following potential problems would the nurse consider when planning care for a client with a persistent occiput posterior position of the fetus? Note: Credit will be given only if all correct choices and no incorrect choices are selected. Select all that apply. 1. Increased fetal mortality 2. Severe perineal lacerations 3. Ceasing of labor progress 4. Fetus born in posterior position 5. Intense back pain during labor

Answer: 2, 3, 4, 5 Explanation: 2. The woman can have third- or fourth-degree perineal laceration or extension of a midline episiotomy. 3. Sometimes labor progress ceases if the fetus fails to rotate to an occiput anterior position. 4. Occiput posterior positions are associated with a higher incidence of vacuum-assisted births. 5. The woman usually experiences intense back pain in the small of her back throughout labor.

The nurse is aware of the different breathing techniques that are used during labor. Why are breathing techniques used during labor? Note: Credit will be given only if all correct choices and no incorrect choices are selected. Select all that apply. 1. They are a form of anesthesia. 2. They are a source of relaxation. 3. They increase the ability to cope with contractions. 4. They are a source of distraction. 5. They increase a woman's pain threshold.

Answer: 2, 3, 4, 5 Explanation: 2. When used correctly, breathing techniques can encourage relaxation. 3. When used correctly, breathing techniques can enhance the ability to cope with uterine contractions. 4. When used correctly, breathing techniques provide some distraction from the pain. 5. When used correctly, breathing techniques increase a woman's pain threshold.

Which of the following symptoms would be an indication of postpartum blues? Note: Credit will be given only if all correct choices and no incorrect choices are selected. Select all that apply. 1. Overeating 2. Anger 3. Mood swings 4. Constant sleepiness 5. Crying

Answer: 2, 3, 5 Explanation: 2. Anger would be a symptom of postpartum blues. 3. Mood swings would be a symptom of postpartum blues. 5. Weepiness and crying would be a symptom of postpartum blues.

Maternal risks of occiput posterior (OP) malposition include which of the following? Note: Credit will be given only if all correct choices and no incorrect choices are selected. Select all that apply. 1. Blood loss greater than 1000 mL 2. Postpartum infection 3. Anal sphincter injury 4. Higher rates of vaginal birth 5. Instrument delivery

Answer: 2, 3, 5 Explanation: 2. Postpartum infection is a maternal risk of OP. 3. Anal sphincter injury is a maternal risk of OP. 5. Instrument delivery is a maternal risk of OP.

Nursing interventions that foster the process of becoming a mother include which of the following? 1. Encouraging detachment from the nurse-patient relationship 2. Promoting maternal-infant attachment 3. Building awareness of and responsiveness to infant interactive capabilities 4. Instruct about promoting newborn independence 5. Preparing the woman for the maternal social role

Answer: 2, 3, 5 Explanation: 2. Promoting maternal-infant attachment is a nursing intervention that fosters the process of becoming a mother. 3. Building awareness of and responsiveness to infant interactive capabilities is a nursing intervention that fosters the process of becoming a mother. 5. Preparing the woman for the maternal social role is a nursing intervention that fosters the process of becoming a mother.

Which physical assessment findings would the nurse consider normal for the postpartum client following a vaginal delivery? Note: Credit will be given only if all correct choices and no incorrect choices are selected. Select all that apply. 1. Elevated blood pressure 2. Fundus firm and midline 3. Moderate amount of lochia serosa 4. Edema and bruising of perineum 5. Inflamed hemorrhoids

Answer: 2, 4 Explanation: 2. A firm fundus that is midline indicates the normal progression of uterine involution. 4. During the early postpartum period, the soft tissue in and around the perineum may appear edematous with some bruising.

In which clinical situations would it be appropriate for an obstetrician to order a labor nurse to perform amnioinfusion? Note: Credit will be given only if all correct choices and no incorrect choices are selected. Select all that apply. 1. Placental abruption 2. Meconium-stained fluid 3. Polyhydramnios 4. Variable decelerations 5. Early decelerations

Answer: 2, 4 Explanation: 2. The physician may order amnioinfusion for meconium-stained fluid. 4. Amnioinfusion is sometimes done to prevent the possibility of variable decelerations.

The nurse determines that a client is carrying her fetus in the vertical (longitudinal) lie. The nurse's judgment should be questioned if the fetal presenting part is which of the following? Note: Credit will be given only for all correct choices and no incorrect choices. Select all that apply. 1. Sacrum 2. Left arm 3. Mentum 4. Left scapula 5. Right scapula

Answer: 2, 4, 5 Explanation: 2. A fetus with an arm presenting is likely in a horizontal lie. 4. A fetus with a left scapula presenting is in a horizontal lie. 5. A fetus with a right scapula presenting is in a horizontal lie.

The nurse is monitoring a client who is receiving an amnioinfusion. Which assessments must the nurse perform to prevent a serious complication? Note: Credit will be given only if all correct choices and no incorrect choices are selected. Select all that apply. 1. Color of amniotic fluid 2. Maternal blood pressure 3. Cervical effacement 4. Uterine resting tone 5. Fluid leaking from the vagina

Answer: 2, 4, 5 Explanation: 2. Blood pressure should be monitored along with other vital signs. 4. The nurse should monitor contraction status (frequency, duration, intensity, resting tone, and associated maternal discomfort). 5. The nurse should continually check to make sure the infused fluid is being expelled from the vagina.

During the first several postpartum weeks, the new mother must accomplish certain physical and developmental tasks, including which of the following? Note: Credit will be given only if all correct choices and no incorrect choices are selected. Select all that apply. 1. Establish a therapeutic relationship with her physician 2. Adapt to altered lifestyles and family structure resulting from the addition of a new member 3. Restore her intellectual abilities 4. Restore physical condition 5. Develop competence in caring for and meeting the needs of her infant

Answer: 2, 4, 5 Explanation: 2. During the first several postpartum weeks, the new mother must adapt to altered lifestyles and family structure resulting from the addition of a new member. 4. During the first several postpartum weeks, the new mother must restore her physical condition. 5. During the first several postpartum weeks, the new mother must develop competence in caring for and meeting the needs of her infant.

) Risk factors for labor dystocia include which of the following? Note: Credit will be given only if all correct choices and no incorrect choices are selected. Select all that apply. 1. Tall maternal height 2. Labor induction 3. Small-for-gestational-age (SGA) fetus 4. Malpresentation 5. Prolonged latent phase

Answer: 2, 4, 5 Explanation: 2. Labor induction is a risk factor of dystocia. 4. Malpresentation is a risk factor of dystocia. 5. Prolonged latent phase is a risk factor of dystocia.

What possible approaches should the nurse use to provide sensitive, holistic nursing care for the mother who is relinquishing her newborn? Note: Credit will be given only if all correct choices and no incorrect choices are selected. Select all that apply. 1. Allow the mother minimal control over the infant. 2. Use active listening strategies to determine the client's needs. 3. Provide only physical care in the early postpartum period. 4. Demonstrate empathy, concern, and compassion. 5. Provide nonjudgmental support and personalized care.

Answer: 2, 4, 5 Explanation: 2. The nurse can support the mother by encouraging her to share her feelings, by listening actively, and by being present for her. 4. The mother who decides to relinquish her baby needs emotional support and validation of her loss. The nurse should demonstrate empathy, concern, and compassion. 5. The nurse needs to acknowledge the significance of the birth mother's experience. The nurse should acknowledge the woman's loss and support her decision.

Amniotomy as a method of labor induction has which of the following advantages? Note: Credit will be given only if all correct choices and no incorrect choices are selected. Select all that apply. 1. The danger of a prolapsed cord is decreased. 2. There is usually no risk of hypertonus or rupture of the uterus. 3. The intervention can cause a decrease in pain. 4. The color and composition of amniotic fluid can be evaluated. 5. The contractions elicited are similar to those of spontaneous labor.

Answer: 2, 4, 5 Explanation: 2. There is usually no risk of hypertonus or rupture of the uterus and this is an advantage of amniotomy. 4. The color and composition of amniotic fluid can be evaluated and this is an advantage of amniotomy. 5. The contractions elicited are similar to those of spontaneous labor and this is an advantage of amniotomy.

Induction of labor is planned for a 31-year-old client at 39 weeks due to insulin-dependent diabetes. Which nursing action is most important? 1. Administer 100 mcg of misoprostol (Cytotec) vaginally every 2 hours. 2. Place dinoprostone (Prepidil) vaginal gel and ambulate client for 1 hour. 3. Begin Pitocin (oxytocin) 4 hours after 50 mcg misoprostol (Cytotec). 4. Prepare to induce labor after administering a tap water enema.

Answer: 3 Explanation: 3. Pitocin should not administered less than 4 hours after the last Cytotec dose.

A client at 40 weeks' gestation is to undergo stripping of the membranes. The nurse provides the client with information about the procedure. Which information is accurate? Note: Credit will be given only if all correct choices and no incorrect choices are selected. Select all that apply. 1. Intravenous administration of oxytocin will be used to initiate contractions. 2. The physician/CNM will insert a gloved finger into the cervical os and rotate the finger 360 degrees. 3. Stripping of the membranes will not cause discomfort, and is usually effective. 4. Labor should begin within 24-48 hours after the procedure. 5. Uterine contractions, cramping, and a bloody discharge can occur after the procedure.

Answer: 2, 4, 5 Explanation: 2. This motion separates the amniotic membranes that are lying against the lower uterine segment and internal os from the distal part of the lower uterine segment. 4. If labor is initiated, it typically begins within 24-48 hours. 5. Uterine contractions, cramping, scant bleeding, and bloody discharge can occur after stripping of the membranes.

An anesthesiologist informs the nurse that a client scheduled for a caesarean section will be having general anesthesia with postoperative self-controlled analgesia. For which clients would a general anesthesia be recommended? Note: Credit will be given only if all correct choices and no incorrect choices are selected. Select all that apply. 1. The client with a history of hypertension 2. The client who has had a lower back fusion 3. The client who is 13 years old 4. The client who is allergic to morphine sulfate 5. The client who has had surgery for scoliosis

Answer: 2, 5 Explanation: 2. Contraindications for epidural block include patients with previous back surgery. 5. Contraindications for epidural block include patients with previous back surgery.

The primary physician orders a narcotic analgesic for a client in labor. Which situations would lead the nurse to hold the medication? Note: Credit will be given only if all correct and no incorrect choices are selected. Select all that apply. 1. Contraction pattern every 3 minutes for 60 seconds 2. Fetal monitor tracing showing late decelerations 3. Client sleeping between contractions 4. Blood pressure 150/90 5. Blood pressure 80/42

Answer: 2, 5 Explanation: 2. Maternal hypotension results in uteroplacental insufficiency in the fetus, which is manifested as late decelerations on the fetal monitoring strip. 5. This would be a contraindication, as a narcotic can lower the blood pressure even more.

The primary care provider is performing a fetal scalp stimulation test. What result would the nurse hope to observe? Note: Credit will be given only if all correct choices and no incorrect choices are selected. Select all that apply. 1. Spontaneous fetal movement 2. Fetal heart acceleration 3. Increase in fetal heart variability 4. Resolution of late decelerations 5. Reactivity associated with the stimulation

Answer: 2, 5 Explanation: 2. The fetal heart rate should accelerate with stimulation. 5. There will be some reaction with the stimulation.

The client demonstrates understanding of the implications for future pregnancies secondary to her classic uterine incision when she states which of the following? 1. "The next time I have a baby, I can try to deliver vaginally." 2. "The risk of rupturing my uterus is too high for me to have any more babies." 3. "Every time I have a baby, I will have to have a cesarean delivery." 4. "I can only have one more baby."

Answer: 3 Explanation: 3. A classic uterine incision is made in the upper uterine segment and is associated with an increased risk of rupture in subsequent pregnancy, labor, and birth. Therefore, subsequent deliveries will be done by cesarean.

The nurse is aware that a fetus that is not in any stress would respond to a fetal scalp stimulation test by showing which change on the monitor strip? 1. Late decelerations 2. Early decelerations 3. Accelerations 4. Fetal dysrhythmia

Answer: 3 Explanation: 3. A fetus that is not experiencing stress responds to scalp stimulation with an acceleration of the FHR.

The client requires vacuum extraction assistance. To provide easier access to the fetal head, the physician cuts a mediolateral episiotomy. After delivery, the client asks the nurse to describe the episiotomy. What does the nurse respond? 1. "The episiotomy goes straight back toward your rectum." 2. "The episiotomy is from your vagina toward the urethra." 3. "The episiotomy is cut diagonally away from your vagina." 4. "The episiotomy extends from your vagina into your rectum."

Answer: 3 Explanation: 3. A mediolateral episiotomy is angled from the vaginal opening toward the buttock. It begins in the midline of the posterior fourchette and extends at a 45-degree angle downward to the right or left.

The nurse is caring for a 15-year-old client who gave birth to her first child yesterday. What action is the best indicator that the nurse understands the parenting adolescent? 1. The client's mother is included in all discussions and demonstrations. 2. The father of the baby is encouraged to change a diaper and give a bottle. 3. The nurse explains the characteristics and cues of the baby when assessing him. 4. A discussion on contraceptive methods is the first topic of teaching.

Answer: 3 Explanation: 3. A newborn physical examination performed at the bedside gives the parent(s) immediate feedback about the newborn's health and demonstrates methods of handling an infant. This action helps the client learn about her baby as an individual and facilitates maternal-infant attachment. This is the highest priority.

On the 3rd day postpartum, a client who is not breastfeeding experiences engorgement. To relieve her discomfort, the nurse should encourage the client to do which of the following? 1. Remove her bra 2. Apply heat to the breasts 3. Apply cold packs to the breasts 4. Use a breast pump to release the milk

Answer: 3 Explanation: 3. Applying cold packs to the breasts relieves discomfort and helps suppress lactation.

A laboring client's obstetrician has suggested amniotomy as a method for inducing labor. Which assessment(s) must be made just before the amniotomy is performed? 1. Maternal temperature, BP, and pulse 2. Estimation of fetal birth weight 3. Fetal presentation, position, and station 4. Biparietal diameter

Answer: 3 Explanation: 3. Before an amniotomy is performed, the fetus is assessed for presentation, position, station, and FHR.

A woman has been in labor for 16 hours. Her cervix is dilated to 3 cm and is 80% effaced. The fetal presenting part is not engaged. The nurse would suspect which of the following? 1. Breech malpresentation 2. Fetal demise 3. Cephalopelvic disproportion (CPD) 4. Abruptio placentae

Answer: 3 Explanation: 3. Cephalopelvic disproportion (CPD) prevents the presenting part from becoming engaged.

The nurse is preparing a client education handout on the differences between false labor and true labor. What information is most important for the nurse to include? 1. True labor contractions begin in the back and sweep toward the front. 2. False labor often feels like abdominal tightening, or "balling up." 3. True labor can be diagnosed only if cervical change occurs. 4. False labor contractions do not increase in intensity or duration.

Answer: 3 Explanation: 3. Cervical change is the only factor that actually distinguishes false from true labor. The contractions of true labor produce progressive dilatation and effacement of the cervix. The contractions of false labor do not produce progressive cervical effacement and dilatation.

The postpartum client is about to go home. The nurse includes which subject in the teaching plan? 1. Replacement of fluids 2. Striae 3. Diastasis of the recti muscles 4. REEDA scale

Answer: 3 Explanation: 3. Diastasis recti abdominis can be improved with exercise and abdominal muscle tone can improve significantly best taught when the mother is receptive to instruction during the postpartum assessment.

The client in early labor asks the nurse what the contractions are like as labor progresses. What would the nurse respond? 1. "In normal labor, as the uterine contractions become stronger, they usually also become less frequent." 2. "In normal labor, as the uterine contractions become stronger, they usually also become less painful." 3. "In normal labor, as the uterine contractions become stronger, they usually also become longer in duration." 4. "In normal labor, as the uterine contractions become stronger, they usually also become shorter in duration."

Answer: 3 Explanation: 3. During the active and transition phases, contractions become more frequent, are longer in duration, and increase in intensity.

Persistent early decelerations are noted. What would the nurse's first action be? 1. Turn the mother on her left side and give oxygen. 2. Check for prolapsed cord. 3. Do nothing. This is a benign pattern. 4. Prepare for immediate forceps or cesarean delivery.

Answer: 3 Explanation: 3. Early decelerations are considered benign, and do not require any intervention.

The laboring client with meconium-stained amniotic fluid asks the nurse why the fetal monitor is necessary, as she finds the belt uncomfortable. Which response by the nurse is most important? 1. "The monitor is necessary so we can see how your labor is progressing." 2. "The monitor will prevent complications from the meconium in your fluid." 3. "The monitor helps us to see how the baby is tolerating labor." 4. "The monitor can be removed, and oxygen given instead."

Answer: 3 Explanation: 3. Electronic fetal monitoring (EFM) provides a continuous tracing of the fetal heart rate (FHR), allowing characteristics of the FHR to be observed and evaluated.

A client dilated to 5 cm has just received an epidural for pain. She complains of feeling lightheaded and dizzy within 10 minutes after the procedure. Her blood pressure was 120/80 before the procedure and is now 80/52. In addition to the bolus of fluids she has been given, which medication is preferred to increase her BP? 1. Epinephrine 2. Terbutaline 3. Ephedrine 4. Epifoam

Answer: 3 Explanation: 3. Ephedrine is the medication of choice to increase maternal blood pressure.

The client with a normal pregnancy had an emergency cesarean birth under general anesthesia 2 hours ago. The client now has a respiratory rate of 30, pale blue nail beds, a pulse rate of 110, and a temperature of 102.6°F, and is complaining of chest pain. The nurse understands that the client most likely is experiencing which of the following? 1. Pulmonary embolus 2. Pneumococcal pneumonia 3. Pneumonitis 4. Gastroesophageal reflux disease

Answer: 3 Explanation: 3. Even when food and fluids have been withheld, the gastric juice produced during fasting is highly acidic and can produce chemical pneumonitis if aspirated. This pneumonitis is known as Mendelson syndrome. The signs and symptoms are chest pain, respiratory distress, cyanosis, fever, and tachycardia. Women undergoing emergency cesarean births appear to be at considerable risk for adverse events.

The client is being admitted to the birthing unit. As the nurse begins the assessment, the client's partner asks why the fetus's heart rate will be monitored. After the nurse explains, which statement by the partner indicates a need for further teaching? 1. "The fetus's heart rate will vary between 110 and 160." 2. "The heart rate is monitored to see whether the fetus is tolerating labor." 3. "By listening to the heart, we can tell the gender of the fetus." 4. "After listening to the heart rate, you will contact the midwife."

Answer: 3 Explanation: 3. Fetal heart rate is not a predictor of gender.

The client is recovering from a delivery that included a midline episiotomy. Her perineum is swollen and sore. Ten minutes after an ice pack is applied, the client asks for another. What is the best response from the nurse? 1. "I'll get you one right away." 2. "You only need to use one ice pack." 3. "You need to leave it off for at least 20 minutes and then reapply." 4. "I'll bring you an extra so that you can change it when you are ready."

Answer: 3 Explanation: 3. For optimal effect, the ice pack should be applied for 20 to 30 minutes and removed for at least 20 minutes before being reapplied.

The nurse is providing preoperative teaching to a client for whom a cesarean birth under general anesthesia is scheduled for the next day. Which statement by the client indicates that she requires additional information? 1. "General anesthesia can be accomplished with inhaled gases." 2. "General anesthesia usually involves administering medication into my IV." 3. "General anesthesia will provide good pain relief after the birth." 4. "General anesthesia takes effect faster than an epidural

Answer: 3 Explanation: 3. General anesthesia provides no pain relief after birth, as regional anesthesia does.

A client's labor has progressed so rapidly that a precipitous birth is occurring. What should the nurse do? 1. Go to the nurse's station and immediately call the physician. 2. Run to the delivery room for an emergency birth pack. 3. Stay with the client and ask auxiliary personnel for assistance. 4. Hold back the infant's head forcibly until the physician arrives for the delivery.

Answer: 3 Explanation: 3. If birth is imminent, the nurse must not leave the client alone.

The nurse determines the fundus of a postpartum client to be boggy. Initially, what should the nurse do? 1. Document the findings. 2. Catheterize the client. 3. Massage the uterine fundus until it is firm. 4. Call the physician immediately.

Answer: 3 Explanation: 3. The nurse would massage the uterine fundus until it is firm by keeping one hand in position and stabilizing the lower portion of the uterus. With one hand used to massage the fundus, the nurse would put steady pressure on the top of the now-firm fundus and to see if she was able to express any clots.

A nurse is caring for several postpartum clients. Which client is demonstrating a problem attaching to her newborn? 1. The client who is discussing how the baby looks like her father 2. The client who is singing softly to her baby 3. The client who continues to touch her baby with only her fingertips 4. The client who picks her baby up when the baby cries

Answer: 3 Explanation: 3. In a progression of touching activities, the mother proceeds from fingertip exploration of the newborn's extremities toward palmar contact with larger body areas and finally to enfolding the infant with the whole hand and arms. If the client continues to touch with only her fingertips, she might not be developing adequate early attachment.

The labor and delivery nurse is preparing a prenatal class about facilitating the progress of labor. Which of the following frequent responses to pain should the nurse indicate is most likely to impede progress in labor? 1. Increased pulse 2. Elevated blood pressure 3. Muscle tension 4. Increased respirations

Answer: 3 Explanation: 3. It is important for the woman to relax each part of her body. Be alert for signs of muscle tension and tightening. Dissociative relaxation, controlled muscle relaxation, and specified breathing patterns are used to promote birth as a normal process.

Two hours after an epidural infusion has begun, a client complains of itching on her face and neck. What should the nurse do? 1. Remove the epidural catheter and apply a Band-Aid to the injection site. 2. Offer the client a cool cloth and let her know the itching is temporary. 3. Recognize that this is a common side effect, and follow protocol for administration of Benadryl. 4. Call the anesthesia care provider to re-dose the epidural catheter.

Answer: 3 Explanation: 3. Itching is a side effect of the medication used for an epidural infusion. Benadryl, an antihistamine, can be administered to manage pruritus.

The nurse is preparing a teaching brochure for Spanish-speaking postpartum clients. Which topics are critical for this population? 1. Baby baths and birth certificates 2. Hygiene practices 3. When and how to contact their healthcare provider 4. Pain-relief options in labor and after birth

Answer: 3 Explanation: 3. Knowing how to contact their healthcare provider at all times is critical so that clients receive appropriate advice and care in case of a problem or emergency. Knowing what to watch for and when to call the healthcare provider also facilitates safety. These are the highest priorities.

The labor and delivery nurse is assigned to four clients in early labor. Which electronic fetal monitoring finding would require immediate intervention? 1. Early decelerations with each contraction 2. Variable decelerations that recover to the baseline 3. Late decelerations with minimal variability 4. Accelerations

Answer: 3 Explanation: 3. Late decelerations are considered a nonreassuring fetal heart rate (FHR) pattern, and therefore require immediate intervention.

The nurse is providing discharge teaching to a woman who delivered her first child 2 days ago. The nurse understands that additional information is needed if the client makes which statement? 1. "I should expect a lighter flow next week." 2. "The flow will increase if I am too active." 3. "My bleeding will remain red for about a month." 4. "I will be able to use a pantiliner in a day or two."

Answer: 3 Explanation: 3. Lochia rubra is dark red in color. It is present for the first 2 to 3 days postpartum. Lochia serosa is a pinkish color and it follows from about the 3rd to the 10th day.

Major perineal trauma (extension to or through the anal sphincter) is more likely to occur if what type of episiotomy is performed? 1. Mediolateral 2. Episiorrhaphy 3. Midline 4. Medical

Answer: 3 Explanation: 3. Major perineal trauma is more likely to occur if a midline episiotomy is performed. The major disadvantage is that a tear of the midline incision may extend through the anal sphincter and rectum.

After noting meconium-stained amniotic fluid and fetal heart rate decelerations, the physician diagnoses a depressed fetus. The appropriate nursing action at this time would be to do what? 1. Increase the mother's oxygen rate. 2. Turn the mother to the left lateral position. 3. Prepare the mother for a higher-risk delivery. 4. Increase the intravenous infusion rate.

Answer: 3 Explanation: 3. Meconium-stained fluid and heart rate decelerations are indications that delivery is considered higher-risk.

After nalbuphine hydrochloride (Nubain) is administered, labor progresses rapidly, and the baby is born less than 1 hour later. The baby shows signs of respiratory depression. Which medication should the nurse be prepared to administer to the newborn? 1. Fentanyl (Sublimaze) 2. Butorphanol tartrate (Stadol) 3. Naloxone (Narcan) 4. Pentobarbital (Nembutal)

Answer: 3 Explanation: 3. Narcan is useful for respiratory depression caused by nalbuphine (Nubain). Respiratory depression in the mother or fetus/newborn can be improved by the administration of naloxone (Narcan), which is a specific antagonist for this agent.

The client at 39 weeks' gestation calls the clinic and reports increased bladder pressure but easier breathing and irregular, mild contractions. She also states that she just cleaned the entire house. Which statement should the nurse make? 1. "You shouldn't work so much at this point in pregnancy." 2. "What you are describing is not commonly experienced in the last weeks." 3. "Your body may be telling you it is going into labor soon." 4. "If the bladder pressure continues, come in to the clinic tomorrow."

Answer: 3 Explanation: 3. One of the premonitory signs of labor is lightening: The fetus begins to settle into the pelvic inlet (engagement). With fetal descent, the uterus moves downward, and the fundus no longer presses on the diaphragm, which eases breathing.

The community nurse is working with a client whose only child is 8 months old. Which statement does the nurse expect the mother to make? 1. "I have a lot more time to myself than I thought I would have." 2. "My confidence level in my parenting is higher than I anticipated." 3. "I am constantly tired. I feel like I could sleep for a week." 4. "My baby likes everyone, and never fusses when she's held by a stranger."

Answer: 3 Explanation: 3. Physical fatigue often affects adjustments and functions of the new mother. The nurse can also provide information about the fatigue that a new mother experiences, strategies to promote rest and sleep at home, and the impact fatigue can have on a woman's emotions and sense of control.

After several hours of labor, the electronic fetal monitor (EFM) shows repetitive variable decelerations in the fetal heart rate. The nurse would interpret the decelerations to be consistent with which of the following? 1. Breech presentation 2. Uteroplacental insufficiency 3. Compression of the fetal head 4. Umbilical cord compression

Answer: 4 Explanation: 4. Variable decelerations occur when there is umbilical cord compression.

A young adolescent is transferred to the labor and delivery unit from the emergency department. The client is in active labor, but did not know she was pregnant. What is the most important nursing action? 1. Determine who might be the father of the baby for paternity testing. 2. Ask the client what kind of birthing experience she would like to have. 3. Assess blood pressure and check for proteinuria. 4. Obtain a Social Services referral to discuss adoption.

Answer: 3 Explanation: 3. Preeclampsia is more common among adolescents than in young adults, and is potentially life-threatening to both mother and fetus. This assessment is the highest priority.

The nurse is supervising a student nurse who is working with a 14-year-old client who delivered her first child yesterday. Which statement indicates that the nursing student understands the particular needs of an adolescent client? 1. "This client will need less teaching, because she will have gotten the right information in school." 2. "Because of her age, this client will require less frequent fundal checks to assess for postpartal hemorrhage." 3. "Because of her age, this client will probably need extra teaching about the terminology for her anatomy." 4. "This client will need to have her grandmother provide day care and help raise the baby."

Answer: 3 Explanation: 3. Some adolescents may not have a working knowledge of their own anatomy and physiology or the related terminology, and they may require special assistance with postpartum hygiene and care.

On the first postpartum day, the nurse teaches the client about breastfeeding. Two hours later, the mother seems to remember very little of the teaching. The nurse understands this memory lapse to be related to which of the following? 1. The taking-hold phase 2. Postpartum hemorrhage 3. The taking-in period 4. Epidural anesthesia

Answer: 3 Explanation: 3. Soon after birth during the taking-in period, the woman tends to be passive and somewhat dependent. She follows suggestions, hesitates about making decisions, and is still rather preoccupied with her needs.

A client who wishes to have an unmedicated birth is in the transition stage. She is very uncomfortable and turns frequently in the bed. Her partner has stepped out momentarily. How can the nurse be most helpful? 1. Talk to the client the entire time. 2. Turn on the television to distract the client. 3. Stand next to the bed with hands on the railing next to the client. 4. Sit silently in the room away from the bed.

Answer: 3 Explanation: 3. Standing next to the bed is supportive without being irritating. The laboring woman fears being alone during labor. The woman's anxiety may be decreased when the nurse remains with her.

An expectant father has been at the bedside of his laboring partner for more than 12 hours. An appropriate nursing intervention would be to do which of the following? 1. Insist that he leave the room for at least the next hour. 2. Tell him he is not being as effective as he was, and that he needs to let someone else take over. 3. Offer to remain with his partner while he takes a break. 4. Suggest that the client's mother might be of more help.

Answer: 3 Explanation: 3. Support persons frequently are reluctant to leave the laboring woman to take care of their own needs. The laboring woman often fears being alone during labor. Even though there is a support person available, the woman's anxiety may be decreased when the nurse remains with her while he takes a break.

A laboring client asks the nurse, "Why does the physician want to use an intrauterine pressure catheter (IUPC) during my labor?" The nurse would accurately explain that the best rationale for using an IUPC is which of the following? 1. The IUPC can be used throughout the birth process. 2. A tocodynamometer is invasive. 3. The IUPC provides more accurate data than does the tocodynamometer. 4. The tocodynamometer can be used only after the cervix is dilated 2 cm.

Answer: 3 Explanation: 3. The IUPC has several benefits over an external tocotransducer or palpation. Because the IUPC is inserted directly into the uterus, it provides near-exact pressure measurements for contraction intensity and uterine resting tone. The increased sensitivity of the IUPC allows for very accurate timing of uterine contractions (UCs).

If the physician indicates a shoulder dystocia during the delivery of a macrosomic fetus, how would the nurse assist? 1. Call a second physician to assist. 2. Prepare for an immediate cesarean delivery. 3. Assist the woman into McRoberts maneuver. 4. Utilize fundal pressure to push the fetus out.

Answer: 3 Explanation: 3. The McRoberts maneuver is thought to change the maternal pelvic angle and therefore reduce the force needed to extract the shoulders, thereby decreasing the incidence of brachial plexus stretching and clavicular fracture.

The student nurse is to perform Leopold maneuvers on a laboring client. Which assessment requires intervention by the staff nurse? 1. The client is assisted into supine position, and the position of the fetus is assessed. 2. The upper portion of the uterus is palpated, then the middle section. 3. After determining where the back is located, the cervix is assessed. 4. Following voiding, the client's abdomen is palpated from top to bottom.

Answer: 3 Explanation: 3. The cervical exam is not part of Leopold maneuvers. Abdominal palpation is the only technique used for Leopold maneuvers.

The nurse is aware that labor and birth will most likely proceed normally when the fetus is in what position? 1. Right-acromion-dorsal-anterior 2. Right-sacrum-transverse 3. Occiput anterior 4. Posterior position

Answer: 3 Explanation: 3. The most common fetal position is occiput anterior. When this position occurs, labor and birth are likely to proceed normally.

The breastfeeding client asks the nurse about appropriate contraception. What is the nurse's best response? 1. "Breastfeeding has many effects on sexual intercourse." 2. "IUDs are easy to use and easy to insert prior to sexual intercourse." 3. "It's possible to get pregnant before your menstrual period returns. Let's talk about some different options for contraception." 4. "Breastfeeding hampers ovulation, so no contraception is needed."

Answer: 3 Explanation: 3. The nurse should discuss the importance of contraception during the early postpartum period and provide information on the advantages and disadvantages of different methods, including special considerations for breastfeeding mothers. The woman's body needs adequate time to heal and recover from the stress of pregnancy and childbirth.

The postpartum nurse is caring for a client who gave birth to full-term twins earlier today. The nurse will know to assess for symptoms of which of the following? 1. Increased blood pressure 2. Hypoglycemia 3. Postpartum hemorrhage 4. Postpartum infection

Answer: 3 Explanation: 3. The nurse will assess for postpartum hemorrhage. Overstretching of uterine muscles with conditions such as multiple gestation, polyhydramnios, or a very large baby may set the stage for slower uterine involution.

The client has been pushing for 2 hours and is exhausted. The physician is performing a vacuum extraction to assist the birth. Which finding is expected and normal? 1. The head is delivered after eight "pop-offs" during contractions. 2. A cephalohematoma is present on the fetal scalp. 3. The location of the vacuum is apparent on the fetal scalp after birth. 4. Positive pressure is applied by the vacuum extraction during contractions.

Answer: 3 Explanation: 3. The parents need to be informed that the caput (chignon) on the baby's head will disappear within 2 to 3 days.

The nurse is observing a new graduate perform a postpartum assessment. Which action requires intervention by the nurse? 1. Asking the client to void and donning clean gloves 2. Listening to bowel sounds and then asking when her last bowel movement occurred 3. Offering the patient pre-medication 2 hours before the assessment 4. Completing the assessment and explaining the results to the client

Answer: 3 Explanation: 3. The patient should be offered premedication 30-45 minutes before assessing the fundus, especially if the patient has had a cesareansection.

A client is admitted to the labor and delivery unit with contractions that are regular, are 2 minutes apart, and last 60 seconds. She reports that her labor began about 6 hours ago, and she had bloody show earlier that morning. A vaginal exam reveals a vertex presenting, with the cervix 100% effaced and 8 cm dilated. The client asks what part of labor she is in. The nurse should inform the client that she is in what phase of labor? 1. Latent phase 2. Active phase 3. Transition phase 4. Fourth stage

Answer: 3 Explanation: 3. The transition phase begins with 8 cm of dilatation, and is characterized by contractions that are closer and more intense.

A client is admitted to the labor and delivery unit with contractions that are 2 minutes apart, lasting 60 seconds. She reports that she had bloody show earlier that morning. A vaginal exam reveals that her cervix is 100 percent effaced and 8 cm dilated. The nurse knows that the client is in which phase of labor? 1. Active 2. Latent 3. Transition 4. Fourth

Answer: 3 Explanation: 3. The transition phase begins with 8 cm to 10 cm of dilatation, and contractions become more frequent, are longer in duration, and increase in intensity.

The nurse is admitting a client to the labor and delivery unit. Which aspect of the client's history requires notifying the physician? 1. Blood pressure 120/88 2. Father a carrier of sickle-cell trait 3. Dark red vaginal bleeding 4. History of domestic abuse

Answer: 3 Explanation: 3. Third-trimester bleeding is caused by either placenta previa or abruptio placentae. Dark red bleeding usually indicates abruptio placentae, which is life-threatening to both mother and fetus.

The nurse is providing education to the new family. Which question by the nurse is best? 1. "Do you know how to give the baby a bath?" 2. "You have diapers and supplies at home, right?" 3. "How have your breastfeedings been going?" 4. "How much formal education do you have?"

Answer: 3 Explanation: 3. This is an open-ended question about an important physiologic issue. A discussion that includes both partners can facilitate an open dialog between them and can provide an opportunity for questions and answers.

Which statement by a new mother 1 week postpartum indicates maternal role attainment? 1. "I don't think I'll ever know what I'm doing." 2. "This baby feels like a real stranger to me." 3. "It works better for me to undress the baby and to nurse in the chair rather than the bed." 4. "My sister took to mothering in no time. Why can't I?"

Answer: 3 Explanation: 3. This statement indicates a stage of maternal role attainment in which the new mother feels comfortable enough to make her own decisions about parenting.

At 1 minute after birth, the infant has a heart rate of 100 beats per minute, and is crying vigorously. The limbs are flexed, the trunk is pink, and the feet and hands are cyanotic. The infant cries easily when the soles of the feet are stimulated. How would the nurse document this infant's Apgar score? 1. 7 2. 8 3. 9 4. 10

Answer: 3 Explanation: 3. Two points each are scored in each of the categories of heart rate, respiratory effort, muscle tone, and reflex irritability. One point is scored in the category of skin color. The total Apgar would be 9.

What is the advantage of a client using a patient-controlled analgesia (PCA) following a cesarean birth? 1. The client receives a bolus of the analgesia when pressing the button. 2. The client experiences pain relief within 30 minutes. 3. The client feels a greater sense of control, and is less dependent on the nursing staff. 4. The client can deliver as many doses of the medication as needed.

Answer: 3 Explanation: 3. Using a special intravenous (IV) pump system, the woman presses a button to self-administer small doses of the medication as needed. For safety, the pump is preset with a time lockout so that the pump cannot deliver another dose until a specified time has elapsed. Women using PCA feel less anxious and have a greater sense of control with less dependence on the nursing staff.

A client is consulting a certified nurse-midwife because she is hoping for a vaginal birth after cesarean (VBAC) with this pregnancy. Which statement indicates that the client requires more information about VBAC? 1. "I can try a vaginal birth because my uterine incision is a low segment transverse incision." 2. "The vertical scar on my skin doesn't mean that the scar on my uterus goes in the same direction." 3. "There is about a 90% chance of giving birth vaginally after a cesarean." 4. "Because my hospital has a surgery staff on call 24 hours a day, I can try a VBAC there."

Answer: 3 Explanation: 3. Women whose previous cesarean was performed because of nonrecurring indications have been reported to have approximately a 60% to 80% chance of success with VBAC.

The nurse is preparing a class for mothers and their partners who have just recently delivered. One topic of the class is infant attachment. Which statement by a participant indicates an understanding of this concept? Note: Credit will be given only if all correct choices and no incorrect choices are selected. Select all that apply. 1. "We should avoid holding the baby too much." 2. "Looking directly into the baby's eyes might frighten him." 3. "Talking to the baby is good because he'll recognize our voices." 4. "Holding the baby so we have direct face-to-face contact is good." 5. "We should only touch the baby with our fingertips for the first month."

Answer: 3, 4 Explanation: 3. Attachment behaviors include cuddling, soothing, and calling the baby by name. 4. Attachment behaviors include holding the baby in the en face position.

Lacerations of the cervix or vagina may be present when bright red vaginal bleeding persists in the presence of a well-contracted uterus. The incidence of lacerations is higher among which of the following childbearing women? Note: Credit will be given only if all correct choices and no incorrect choices are selected. Select all that apply. 1. Over the age of 35 2. Have not had epidural block 3. Have had an episiotomy 4. Have had a forceps-assisted or vacuum-assisted birth 5. Nulliparous

Answer: 3, 4 Explanation: 3. The incidence of lacerations is higher among childbearing women who undergo an episiotomy. 4. The incidence of lacerations is higher among childbearing women who undergo forceps-assisted or vacuum-assisted birth.

The labor and birth nurse is admitting a client. The nurse's assessment includes asking the client whom she would like to have present for the labor and birth, and what the client would prefer to wear. The client's partner asks the nurse the reason for these questions. What would the nurse's best response be? Note: Credit will be given only if all correct choices and no incorrect choices are selected. Select all that apply. 1. "These questions are asked of all women. It's no big deal." 2. "I'd prefer that your partner ask me all the questions, not you." 3. "A client's preferences for her birth are important for me to understand." 4. "Many women have beliefs about childbearing that affect these choices." 5. "I'm gathering information that the nurses will use after the birth."

Answer: 3, 4 Explanation: 3. The nurse incorporates the family's expectations into the plan of care to be culturally appropriate and to facilitate the birth. 4. The nurse incorporates the family's expectations into the plan of care to be culturally appropriate and to facilitate the birth.

The nurse is caring for laboring clients. Which women are experiencing problems related to a critical factor of labor? Note: Credit will be given only for all correct choices and no incorrect choices. Select all that apply. 1. Woman at 7 cm, fetus in general flexion 2. Woman at 3 cm, fetus in longitudinal lie 3. Woman at 4 cm, fetus with transverse lie 4. Woman at 6 cm, fetus at -2 station, mild contractions 5. Woman at 5 cm, fetal presenting part is right shoulder

Answer: 3, 4, 5 Explanation: 3. A transverse lie occurs when the cephalocaudal axis of the fetal spine is at a right angle to the woman's spine and is associated with a shoulder presentation and can lead to complications in the later stages of labor. 4. Station refers to the relationship of the presenting part to an imaginary line drawn between the ischial spines of the maternal pelvis. If the presenting part is higher than the ischial spines, a negative number is assigned, noting centimeters above zero station. A -2 station is high in the pelvis. Contractions should be strong to cause fetal descent. Mild contractions will not move the baby down or open the cervix. This client is experiencing a problem between the maternal pelvis and the presenting part. 5. When the fetal shoulder is the presenting part, the fetus is in a transverse lie and the acromion process of the scapula is the landmark. This type of presentation occurs less than 1% of the time. This client is experiencing a problem between the maternal pelvis and the presenting part.

A woman is scheduled to have an external version for a breech presentation. The nurse carefully reviews the client's chart for contraindications to this procedure, including which of the following? Note: Credit will be given only if all correct choices and no incorrect choices are selected. Select all that apply. 1. Station -2 2. 38 weeks' gestation 3. Abnormal fetal heart rate and tracing 4. Previous cesarean section 5. Rupture of membranes

Answer: 3, 4, 5 Explanation: 3. An abnormal fetal heart rate or tracing would be a contraindication to performing a version. A nonreassuring FHR pattern might indicate that the fetus is already stressed and other action needs to be taken. 4. A previous cesarean is a contraindication for version. 5. Rupture of membranes is a contraindication for version because of insufficient amniotic fluid.

An abbreviated systematic physical assessment of the newborn is performed by the nurse in the birthing area to detect any abnormalities. Normal findings would include which of the following? Note: Credit will be given only if all correct choices and no incorrect choices are selected. Select all that apply. 1. Skin color: Body blue with pinkish extremities 2. Umbilical cord: two veins and one artery 3. Respiration rate of 30-60 irregular 4. Temperature of above 36.5°C (97.8°F) 5. Sole creases that involve the heel

Answer: 3, 4, 5 Explanation: 3. Normal findings would include a respiration rate of 30-60 irregular. 4. Normal findings would include temperature of above 36.5°C (97.8°F). 5. Normal findings would include sole creases that involve the heel.

True postterm pregnancies are frequently associated with placental changes that cause a decrease in the uterine-placental-fetal circulation. Complications related to alternations in placenta functioning include which of the following? Note: Credit will be given only if all correct choices and no incorrect choices are selected. Select all that apply. 1. Increased fetal oxygenation 2. Increased placental blood supply 3. Reduced nutritional supply 4. Macrosomia 5. Risk of shoulder dystocia

Answer: 3, 4, 5 Explanation: 3. Reduced nutritional supply is a complication related to alternations in placenta functioning. 4. Macrosomia is a complication related to alternations in placenta functioning. 5. Risk of shoulder dystocia is a complication

The nurse is planning discharge teaching for a postpartum woman. What information recommendations should the woman receive before being discharged? Note: Credit will be given only if all correct choices and no incorrect choices are selected. Select all that apply. 1. To abstain from sexual intercourse for 6 months 2. To avoid showers for 4 weeks 3. To avoid overexertion 4. To practice postpartum exercises 5. To obtain adequate rest

Answer: 3, 4, 5 Explanation: 3. The client should avoid overexertion. 4. The client should receive information and instruction on postpartum exercises. 5. The client should receive information on the need for adequate rest.

Five clients are in active labor in the labor unit. Which women should the nurse monitor carefully for the potential of uterine rupture? Note: Credit will be given only if all correct choices and no incorrect choices are selected. Select all that apply. 1. Age 15, in active labor 2. Age 22, with eclampsia 3. Age 25, last delivery by cesarean section 4. Age 32, first baby died during labor 5. Age 27, last delivery 11 months ago

Answer: 3, 5 Explanation: 3. A woman who has had a previous cesarean section is at risk for uterine rupture. 5. A woman who does not have at least 18 months between deliveries is at greater risk for uterine rupture.

The midwife performs a vaginal exam and determines that the fetal head is at a -2 station. What does this indicate to the nurse about the birth? 1. The birth is imminent. 2. The birth is likely to occur in 1-2 hours. 3. The birth will occur later in the shift. 4. The birth is difficult to predict.

Answer: 4 Explanation: 4. A -2 station means that the fetus is 2 cm above the ischial spines. The ischial spines as a landmark have been designated as zero station. If the presenting part is higher than the ischial spines, a negative number is assigned, noting centimeters above zero station. With the fetus's head that high in the pelvis, it is difficult to predict when birth will occur.

The nurse is explaining induction of labor to a client. The client asks what the indications for labor induction are. Which of the following should the nurse include when answering the client? 1. Suspected placenta previa 2. Breech presentation 3. Prolapsed umbilical cord 4. Hypertension

Answer: 4 Explanation: 4. A client with hypertension is appropriate for labor induction.

The nurse is performing a postpartum assessment on a newly delivered client. When checking the fundus, there is a gush of blood. The client asks why that is happening. What is the nurse's best response? 1. "We see this from time to time. It's not a big deal." 2. "The gush is an indication that your fundus isn't contracting." 3. "Don't worry. I'll make sure everything is fine." 4. "Blood pooled in the vagina while you were in bed."

Answer: 4 Explanation: 4. A gush of blood when a fundal massage is undertaken may occur because of normal pooling of blood in vagina when the woman lies down to rest or sleep.

The client has delivered a 4200 g fetus. The physician performed a midline episiotomy, which extended into a third-degree laceration. The client asks the nurse where she tore. Which response is best? 1. "The episiotomy extended and tore through your rectal mucosa." 2. "The episiotomy extended and tore up near your vaginal mucous membrane." 3. "The episiotomy extended and tore into the muscle layer." 4. "The episiotomy extended and tore through your anal sphincter."

Answer: 4 Explanation: 4. A third degree laceration includes the anal sphincter.

After inserting prostaglandin gel for cervical ripening, what should the nurse do? 1. Apply an internal fetal monitor. 2. Insert an indwelling catheter. 3. Withhold oral intake and start intravenous fluids. 4. Place the client in a supine position with a right hip wedge.

Answer: 4 Explanation: 4. After the gel, intravaginal insert, or tablet is inserted, the woman is instructed to remain lying down with a rolled blanket or hip wedge under her right hip to tip the uterus slightly to the left for the first 30 to 60 minutes to maintain the cervical ripening agent in place.

The nurse is preparing to assess a laboring client who has just arrived in the labor and birth unit. Which statement by the client indicates that additional education is needed? 1. "You are going to do a vaginal exam to see how dilated my cervix is." 2. "The reason for a pelvic exam is to determine how low in the pelvis my baby is." 3. "When you check my cervix, you will find out how thinned out it is." 4. "After you assess my pelvis, you will be able to tell when I will deliver."

Answer: 4 Explanation: 4. An experienced labor and birth nurse can estimate the time of delivery based on the cervix, fetal position, station, and contraction pattern. However, during a pelvic exam, no information is obtained about contractions. The nurse will not have enough information following the cervical exam to estimate time of birth.

In the operating room, a client is being prepped for a cesarean delivery. The doctor is present. What is the last assessment the nurse should make just before the client is draped for surgery? 1. Maternal temperature 2. Maternal urine output 3. Vaginal exam 4. Fetal heart tones

Answer: 4 Explanation: 4. Ascertain fetal heart rate (FHR) before surgery and during preparation because fetal hypoxia can result from aortocaval compression.

The nurse is preparing to receive a newly delivered client. The client is a young single mother who is relinquishing custody of her newborn through an open adoption. What action is most important? 1. Assign the client a room on the GYN surgical floor instead of on the postpartum floor. 2. Prepare to complete teaching in time for discharge at 24 hours post-delivery. 3. Make an effort not to bring up the topic of the baby, and discuss the mother's health instead. 4. Ask the client how much contact she would like with the baby, and whether she wants to feed it.

Answer: 4 Explanation: 4. Assessing the birth mother's preferences by respectfully asking questions and making no assumptions facilitates a more positive experience.

Toward the end of the first stage of labor, a pudendal block is administered transvaginally. What will the nurse anticipate the client's care will include? 1. Monitoring for hypotension every 15 minutes 2. Monitoring FHR every 15 minutes 3. Monitoring for bladder distention 4. No additional assessments

Answer: 4 Explanation: 4. Because a pudendal block is done using a local anesthetic, there is no need for additional monitoring of the mother or the fetus.

As compared with admission considerations for an adult woman in labor, the nurse's priority for an adolescent in labor would be which of the following? 1. Cultural background 2. Plans for keeping the infant 3. Support persons 4. Developmental level

Answer: 4 Explanation: 4. Because her cognitive development is incomplete, the younger adolescent may have fewer problem-solving capabilities. The very young woman needs someone to rely on at all times during labor. She may be more childlike and dependent than older teens.

The nurse is caring for a laboring client. A cervical exam indicates 8 cm dilation. The client is restless, frequently changing position in an attempt to get comfortable. Which nursing action is most important? 1. Leave the client alone so she can rest. 2. Ask the family to take a coffee-and-snack break. 3. Encourage the client to have an epidural for pain. 4. Reassure the client that she will not be left alone.

Answer: 4 Explanation: 4. Because the client is in the transitional phase of the first stage of labor, she will not want to be left alone; staying with the client and reassuring her that she will not be alone are the highest priorities at this time.

The nurse is caring for a client in active labor. The membranes spontaneously rupture, with a large amount of clear amniotic fluid. Which nursing action is most important to undertake at this time? 1. Assess the odor of the amniotic fluid. 2. Perform Leopold maneuvers. 3. Obtain an order for pain medication. 4. Complete a sterile vaginal exam.

Answer: 4 Explanation: 4. Checking the cervix will determine whether the cord prolapsed when the membranes ruptured. The nurse would assess for prolapsed cord via vaginal examination.

The hospital is developing a new maternity unit. What aspects should be included in the planning of the new unit to best promote family wellness? 1. Normal newborn nursery centrally located to all client rooms 2. A kitchen with a refrigerator stocked with juice and sandwiches 3. Small, cozy rooms with a client bed and rocking chair 4. A nursing care model based on providing couplet care

Answer: 4 Explanation: 4. Couplet care, which is care of both the mother and her baby, is an important part of the family-centered care approach, in which the infant remains at the mother's bedside and both are cared for by the same nurse.

To actively involve the postpartal client during discharge teaching, the postpartum nurse applies which learning principle? 1. Reprints of magazine articles 2. Classroom lectures 3. Audiotapes 4. Interactive nurse-patient relationships

Answer: 4 Explanation: 4. Effective parent learning requires precise timing of teaching, as well as choice of a teaching method that is effective for the family, such as DVDs and return demonstration. Content on self-care, infant care, and anticipatory guidance is important.

The client delivered her first child vaginally 7 hours ago. She has not voided since delivery. She has an IV of lactated Ringer's solution running at 100 mL/hr. Her fundus is firm and to the right of midline. What is the best nursing action? 1. To massage the fundus vigorously 2. To assess the client's pain level 3. To increase the rate of the IV 4. To assist the client to the bathroom

Answer: 4 Explanation: 4. Emptying the bladder is the top priority.

A client is having contractions that last 20-30 seconds and that are occurring every 8-20 minutes. The client is requesting something to help relieve the discomfort of contractions. What should the nurse suggest? 1. That a mild analgesic be administered 2. An epidural 3. A local anesthetic block 4. Nonpharmacologic methods of pain relief

Answer: 4 Explanation: 4. For this pattern of labor, nonpharmacologic methods of pain relief should be suggested. These can include back rubs, providing encouragement, and clean linens.

The nurse is beginning the postpartum teaching of a mother who has given birth to her first child. What aspect of teaching is most important? 1. Describe the likely reaction of siblings to the new baby. 2. Discuss adaptation to grandparenthood by her parents. 3. Determine whether father-infant attachment is taking place. 4. Assist the mother in identifying the baby's behavior cues.

Answer: 4 Explanation: 4. Helping the mother to identify her baby's behavior cues facilitates the acquaintance phase of maternal-infant attachment.

The nurse examines the client's placenta and finds that the umbilical cord is inserted at the placental margin. The client comments that the placenta and cord look different than they did for her first two births. The nurse should explain that this variation in placenta and cord is called what? 1. Placenta accreta 2. Circumvallate placenta 3. Succenturiate placenta 4. Battledore placenta

Answer: 4 Explanation: 4. In battledore placenta, the umbilical cord is inserted at or near the placental margin.

The nurse has presented a session on pain relief options to a prenatal class. Which statement indicates that additional teaching is needed? 1. "An epidural can be continuous or can be given in one dose." 2. "A spinal is usually used for a cesarean birth." 3. "Pudendal blocks are effective when a vacuum is needed." 4. "Local anesthetics provide good labor pain relief."

Answer: 4 Explanation: 4. Local anesthetics are not used for labor pain relief. They are used prior to episiotomy and for laceration repair.

The client is at 42 weeks' gestation. Which order should the nurse question? 1. Obtain biophysical profile today. 2. Begin nonstress test now. 3. Schedule labor induction for tomorrow. 4. Have the client return to the clinic in 1 week.

Answer: 4 Explanation: 4. Many practitioners use twice-weekly testing providing the amniotic fluid level is normal. One week is too long a period between assessments.

The client has stated that she wants to avoid an epidural and would like an unmedicated birth. Which nursing action is most important for this client? 1. Encourage the client to vocalize during contractions. 2. Perform vaginal exams only between contractions. 3. Provide a CD of soft music with sounds of nature. 4. Offer to teach the partner how to massage tense muscles.

Answer: 4 Explanation: 4. Massage is helpful for many clients, especially during latent and active labor. Massage can increase relaxation and therefore decrease tension and pain.

A woman is in labor. The fetus is in vertex position. When the client's membranes rupture, the nurse sees that the amniotic fluid is meconium-stained. What should the nurse do immediately? 1. Change the client's position in bed. 2. Notify the physician that birth is imminent. 3. Administer oxygen at 2 liters per minute. 4. Begin continuous fetal heart rate monitoring.

Answer: 4 Explanation: 4. Meconium-stained amniotic fluid is an abnormal fetal finding, and is an indication for continuous fetal monitoring.

The nurse assesses the postpartum client who has not had a bowel movement by the third postpartum day. Which nursing intervention would be appropriate? 1. Encourage the new mother, saying, "It will happen soon." 2. Instruct the client to eat a low-fiber diet. 3. Decrease fluid intake. 4. Obtain an order for a stool softener.

Answer: 4 Explanation: 4. Obtaining an order for a stool softener is the correct intervention by the third day. In resisting or delaying the bowel movement, the woman may cause increased constipation and more pain when elimination finally occurs.

) The nurse is caring for a client who recently emigrated from a Southeast Asian country. The mother has been resting since the birth, while her sister has changed the diapers and fed the infant. What is the most likely explanation for this behavior? 1. The client is not attaching to her infant appropriately. 2. The client is not going to be a good mother, and the baby is at risk. 3. The client has no mother present to role-model behaviors. 4. The client is exhibiting normal behavior for her culture.

Answer: 4 Explanation: 4. Rest, seclusion, and dietary restraint practices in many traditional non-Western cultures (South Asian groups) are designed to assist the woman and her baby during postpartum vulnerable periods.

The nurse has just palpated a laboring woman's contractions. The uterus cannot be indented during a contraction. What would the intensity of these contractions best be characterized as? 1. Weak 2. Mild 3. Moderate 4. Strong

Answer: 4 Explanation: 4. Strong intensity exists when the uterine wall cannot be indented.

During the fourth stage of labor, the client's assessment includes a BP of 110/60, pulse 90, and the fundus is firm midline and halfway between the symphysis pubis and the umbilicus. What is the priority action of the nurse? 1. Turn the client onto her left side. 2. Place the bed in Trendelenburg position. 3. Massage the fundus. 4. Continue to monitor.

Answer: 4 Explanation: 4. The client's assessment data are normal for the fourth stage of labor, so monitoring is the only action necessary. During the fourth stage of labor, the mother experiences a moderate drop in both systolic and diastolic blood pressure, increased pulse pressure, and moderate tachycardia.

During a postpartum examination of a client who delivered an 8-pound newborn 6 hours ago, the following assessment findings are noted: fundus firm and at the umbilicus, and moderate lochia rubra with a steady trickle of blood from the vagina. What is the assessment finding that would necessitate follow-up? 1. Firm fundus 2. Fundus at the umbilical level 3. Moderate lochia rubra 4. Steady trickle of blood

Answer: 4 Explanation: 4. The continuous seepage of blood is more consistent with cervical or vaginal lacerations. Lacerations should be suspected if the uterus is firm and of expected size and if no clots can be expressed. This finding would indicate a follow-up.

During a maternal assessment, the nurse determines the fetus to be in a left occiput anterior (LOA) position. Auscultation of the fetal heart rate should begin in what quadrant? 1. Right upper quadrant 2. Left upper quadrant 3. Right lower quadrant 4. Left lower quadrant

Answer: 4 Explanation: 4. The fetal heart rate (FHR) is heard most clearly at the fetal back. Thus, in a cephalic presentation, the FHR is best heard in the lower quadrant of the maternal abdomen.

While caring for a client in labor, the nurse notices during a vaginal exam that the fetus's head has rotated internally. What would the nurse expect the next set of cardinal movements for a fetus in a vertex presentation to be? 1. Flexion, extension, restitution, external rotation, and expulsion 2. Expulsion, external rotation, and restitution 3. Restitution, flexion, external rotation, and expulsion 4. Extension, restitution, external rotation, and expulsion

Answer: 4 Explanation: 4. The fetus changes position in the following order: descent, flexion, internal rotation, extension, restitution, external rotation, and expulsion.

A client has just been admitted for labor and delivery. She is having mild contractions lasting 30 seconds every 15 minutes. The client wants to have a medication-free birth. When discussing medication alternatives, the nurse should be sure the client understands which of the following? 1. In order to respect her wishes, no medication will be given. 2. Pain relief will allow a more enjoyable birth experience. 3. The use of medications allows the client to rest and be less fatigued. 4. Maternal pain and stress can have a more adverse effect on the fetus than would a small amount of analgesia.

Answer: 4 Explanation: 4. The nurse can explain to the couple that, although pharmacologic agents do affect the fetus, so do the pain and stress experienced by the laboring mother. If the woman's pain and anxiety are more than she can cope with, the adverse physiologic effects on the fetus may be as great as would occur with the administration of a small amount of an analgesic agent. Once the effects are explained, however, it is still the client's choice whether to receive medication.

To assess the healing of the uterus at the placental site, what does the nurse assess? 1. Lab values 2. Blood pressure 3. Uterine size 4. Type, amount, and consistency of lochia

Answer: 4 Explanation: 4. The type, amount, and consistency of lochia determine the state of healing of the placental site, and a progressive color change from bright red at birth to dark red to pink to white or clear should be observed.

Prior to receiving lumbar epidural anesthesia, the nurse would anticipate placing the laboring client in which position? 1. On her right side in the center of the bed with her back curved 2. Lying prone with a pillow under her chest 3. On her left side with the bottom leg straight and the top leg slightly flexed 4. Sitting on the edge of the bed

Answer: 4 Explanation: 4. The woman is positioned on her left or right side, at the edge of the bed with the assistance of the nurse, with her legs slightly flexed, or she is asked to sit on the edge of the bed.

Why is it important for the nurse to assess the bladder regularly and encourage the laboring client to void frequently? 1. A full bladder impedes oxygen flow to the fetus. 2. Frequent voiding prevents bruising of the bladder. 3. Frequent voiding encourages sphincter control. 4. A full bladder can impede fetal descent.

Answer: 4 Explanation: 4. The woman should be encouraged to void because a full bladder can interfere with fetal descent. If the woman is unable to void, catheterization may be necessary.

The nurse is completing discharge teaching for a client who delivered 2 days ago. Which statement by the client indicates that further information is required? 1. "Because I have a midline episiotomy, I should keep my perineum clean." 2. "I can use an ice pack to relieve some the pain from the episiotomy." 3. "I can take ibuprofen (Motrin) when my perineum starts to hurt." 4. "The tear I have through my rectum is unrelated to my episiotomy."

Answer: 4 Explanation: 4. This statement is incorrect. The major disadvantage is that a tear of the midline incision may extend through the anal sphincter and rectum.

How does the nurse assess for Homans' sign? 1. Extending the foot and inquiring about calf pain. 2. Extending the leg and inquiring about foot pain. 3. Flexing the knee and inquiring about thigh pain. 4. Dorsiflexing the foot and inquiring about calf pain.

Answer: 4 Explanation: 4. To assess for thrombophlebitis, the nurse should have the woman stretch her legs out, with the knees slightly flexed and the legs relaxed. The nurse then grasps the foot and dorsiflexes it sharply. If pain is elicited, the nurse notifies the physician/CNM that the woman has a positive Homans' sign. The pain is caused by inflammation of a vessel.

The nurse is orienting a new graduate nurse to the labor and birth unit. Which statement indicates that teaching has been effective? 1. "When a client arrives in labor, a urine specimen is obtained by catheter to check for protein and ketones." 2. "When a client arrives in labor, she will be positioned supine to facilitate a normal blood pressure." 3. "When a client arrives in labor, her prenatal record is reviewed for indications of domestic abuse." 4. "When a client arrives in labor, a vaginal exam is performed unless birth appears to be imminent."

Answer: 4 Explanation: 4. Unless delivery seems imminent because the client is bearing down or contractions are very close and strong, the vaginal exam is performed after the vital signs are obtained.

The client is having fetal heart rate decelerations. An amnioinfusion has been ordered for the client to alleviate the decelerations. The nurse understands that the type of decelerations that will be alleviated by amnioinfusion is which of the following? 1. Early decelerations 2. Moderate decelerations 3. Late decelerations 4. Variable decelerations

Answer: 4 Explanation: 4. When cord compression is suspected, amnioinfusion (AI) may be considered. AI helps to prevent the possibility of variable decelerations by increasing the volume of amniotic fluid.

During the initial intrapartal assessment of a client in early labor, the nurse performs a vaginal examination. The client's partner asks why this pelvic exam needs to be done. The nurse should explain that the purpose of the vaginal exam is to obtain information about which of the following? Note: Credit will be given only if all correct choices and no incorrect choices are selected. Select all that apply. 1. Uterine contraction pattern 2. Fetal position 3. Presence of the mucous plug 4. Cervical dilation and effacement 5. Presenting part

Answer: 4, 5 Explanation: 4. The vaginal examination of a laboring client obtains information about the station of the presenting part and the dilation and effacement of the cervix. 5. The vaginal examination of a laboring client obtains information about the fetal presenting part.

A woman who is 40 weeks pregnant calls the labor suite to ask whether she should be evaluated. Which statements by the client indicate she is likely in labor? Note: Credit will be given only for all correct choices and no incorrect choices. Select all that apply. 1. "The contractions are 5-20 minutes apart." 2. "I had pink discharge on the toilet paper." 3. "I have had cramping for the past 3-4 hours." 4. "The contractions start in my back and then go to my abdomen and are very intense." 5. "The contractions hurt more when I walk."

Answer: 4, 5 Explanation: 4. This is a sign of true labor. The contractions increase in duration and intensity and begin in the back and radiate around to the abdomen. 5. It is a sign of true labor when the client is unable to walk during the contraction.


Kaugnay na mga set ng pag-aaral

Chapter 13 fundamentals of urine and body fluids

View Set

Chemistry Honors final exam review (part 2)

View Set

Chapter 40 - Legal Issues, Quality Assurance, and Infection Prevention

View Set

SPC Level 2 Exam 3- Acute Coronary Syndrome Adaptive Quiz

View Set

Ch 12: Stress, Coping, and Health

View Set

bus5 187 -Global Dimension Business (chapter 1 )

View Set